475.
1-Pathogen commonly implicated in acute suppurative otitis media
inclued, excipet:
A. streptococcus pyogenes.
B. hemophilus influenzae.
C. moraxella catarrhalis.
D. pseudomoneus aerogenosa.
2. Which test reflects the vestibulo-spinal function ?
A. Headshake nystagmus.
B. Electronystagmography.
C. Rotatary chair testing.
D. Tandem walking test.
E. Positional test.
3. Following is true regarding rhinitis medicamentosa
A. It is caused by steroid drops
B. There is atrophy of inferior turbinate
C. It is caused by oral pseudoephidrine
D. Septoplasty is treatment of choice
E. It is caused by vasoconstrictive drops
4. Following are causes of epistaxis except
A. Trauma
B. Ostler
,s disease
C. Nasal polyp
D. Septal perforation
E. Idiopathic
5. A patient has been put on IV heparin for pulmonary embolism .He
started
nose bleed . His PT is 50 seconds. His treatment is
A. Give adrenaline
B. Give normal saline
C. Give protamine sulphate
D. Give blood transfusion
E. Give transamine
476. 6-A young boy of ten years age went for swimming. Next day he
developed nasal congestion, fever and headache .He deteriorated over
next twenty four hours and developed redness and edema of both
eyelids on left side. His likely diagnosis is :
A. Acute maxillary sinusitis
B. Cavernous sinus thrombosis
C. Meningitis
D. Erysipelas
E. Acute ethmoiditis
7. Allergic rhinitis
A. Is a type 1 IgE mediated hypersensitivity reaction
B. Is a type 2 IgM mediated reaction
C. IgA antibodies are produced in desensitization process which block
IgE
D. Total serum IgE levels have to be checked before starting
immunotherapy
E. Is very common in old age.
8. A young woman was on contraceptive pills due to which she
developed deep vein thrombosis .She was placed on warfarin .She
started bleeding from nose. The best test to check her coagulation
status is :
A. Bleeding time
B. Clotting time
C. Platelet count
D. Activated partial thromboplastin time
E. Hess test.
9. A young boy was involved in road traffic accident and broke his nose.
Next day he noticed a watery fluid coming from his left nostril.
Which test will you do to confirm the nature of fluid.
A. Blood sugar
B. Blood sugar in the fluid collected from nose
C. Leukocyte count in fluid
D. Do biochemical analysis of fluid and match the findings with findings
of specimen collected by lumbar puncture
E. Protein levels in nasal fluid
477. 10-A 40 year old man who is on steroids developed boil in the nose
.Two days later he developed swelling of eye lids on one side with
chemosis and opthalmoplegia of extraoccular muscles . He also has
fever. Which investigation will clinch the diagnosis .
A. Total leukocyte count
B. ESR
C. X-ray Para nasal sinuses
D. MRI scan
E. Quickenstead
,s test
11. A young girl has developed saddling of nose after trauma .The best
thing to do is
A. Septoplasty
B. Tip plasty
C. SMR
D. Augmentation rhinoplasty
E. septorhinoplasty
12. Following are measures for house dust mite avoidance in allergic
rhinitis except.
A. Do dry dusting
B. Avoid carpets in bed room
C. Use polythine bedcovers for mattress
D. Wash bed linen in boiling water
E. Do not do dusting in the morning.
13. The skeleton of nasal tip is formed by.
A. Caudal end of the septal cartilage
B. Lower lateral cartilage
C. Upper lateral cartilage
D. Sesamoid cartilage
E. Hyaline cartilage
14. A two year old child is having unilateral nasal discharge which is foul
smelling and purulent the likely diagnosis is.
A. Adenocarcinoma of nose
B. Foreign body nose
C. Sinusitis
D. Nasal polyp
E. Rhinitis
478. 15-The best treatment for recurrent nasal polyps is?
A. Intranasal polypectomy
B. Transantral ethmoidectomy
C. Functional endoscopic sinus surgery
D. Steroids
E. Transantral ethmoidectomy with postoperative inhaled steroids.
16. Major portion of nasal cavity is lined by?
A. Columnar epithelium (respiratory type)
B. Olfactory epithelium
C. Squamous epithelium
D. Skin
E. Cuboidal epithelium
17. Following structures open into middle meatus?
A. Frontal sinus
B. Nasolacrimal duct
C. Sphenoid sinus
D. Posterior ethmoid cells
E. Hiatus semilunaris
18. The middle turbinate?
A. Is part of ethmoid bone
B. Is part of maxilla
C. Is a separate bone
D. It is only mucosal structure
E. It contains cartilage and bone covered with mucosa
19. CSF formation is a process of:
a) Transudation.
b) Exudation.
c) Ultrafiltration.
d) Filtration.
e) Translocation.
479. 20-The patient has developed traumatic CSF rhinorrhoea .The best way
to localize the site of leak is?
A. CT scan
B. MRI scan
C. Radioisotope scan
D. Use florescence dye intrathecally and examine nose with blue
nasoendoscope
E. Examine nose with endoscope
21. Which cranial nerve lies in cavernous sinus as lateral relation of
sphenoid sinus?
A. Trochlear nerve
B. Facial nerve
C. Vidian nerve
D. Sphenoplatine nerve
E. Mandibular division of maxillary nerve
22. Skeletal support to columella is provided by?
A. Caudal end of quadrilateral cartilage
B. Middle crura of allay cartilage
C. Medial crura of allar cartilage
D. Medial crural feet of allar cartilage
E. Anterior nasal spine.
23. The floor of maxillary sinus is closely related to?
A. Canine teeth
B. Molar teeth
C. Premolar teeth
D. Second premolar and first molar
E. All of above
24. A young boy was involved in road traffic accident . He developed
vertigo when he tried to move about . On examination he had blue ear
drum and facial palsy on right side. Audiogram done two weeks later
confirmed sensorineural hearing loss. The likely diagnosis is?
A. Transverse fracture of temporal bone
B. Longitudinal fracture of temporal bone
C. Ossicular chain disruption
D. Disruption of spiral ganglion
E. Brain contusion
480
25. A two year old child is suffering from acute otitis media. His pain is
not relieved in spite of adequate treatment .The best treatment in this
situation is?
A. Change antibiotic
B. Increase analgesic dose
C. Do hot fomentation
D. Give lignocaine ear drops
E. Do myringotomy.
26. Which of the following is not a clinical feature of baurotraumatic
otitis media?
A. Otalgia
B. Deafness
C. Tinnitus
D. Blue ear drum
E. Purulent ear discharge
27. Which of the following is not included in the treatment of
perichondritis of pinna?
A. Local antibiotics
B. Systemic antibiotics
C. Incision and drainage
D. Removal of necrotic cartilage
E. Subsequent plastic surgery
28. A thirty year old lady had flue which was followed by severe otalgia
and tinnitus. One day later she had blood stained discharge from ears
and on otoscopy had reddish vesicles on ear drum. She is suffering
from :
A. Malignant otitis externa
B. Otitis externa hemorrhagica
C. Ramsay-Hunt syndrome
D. Hemotympanum
E. Acute otitis media
481. 29-A two year child is very slow in developing language. On otoscopy
he has dull ear drum. Investigation of choice is?
A. Audiogram
B. X-ray mastoids
C. Tympanogram
D. Serum bilirubin
E. Nystagmogram
30. Following are characteristics of discharge in unsafe chronic
supurative otitis media except?
A. Thick discharge
B. Scanty discharge
C. Odorless discharge
D. Blood stained discharge
E. Purulent discharge
31. A 20 year old boy had long standing foul smelling ear discharge.
Recently he has developed otalgia, fever, headache and vomiting .The
likely diagnosis is?
A. Brain abscess
B. Bezolds abscess
C. Perilabyrinthine fistula
D. Otitis externa
E. Meningitis.
32. Which of the following is not a cause of hoarseness?
A. Carcinoma of right bronchus
B. Acute laryngitis
C. Thyroidectomy
D. Hypothyroidism
E. Carcinoma of larynx
33. Which of the following is not true regarding tracheobronichial
foreign body in the children?
A. The child presents with breathing difficulty
B. The foreign body usually lodges in left bronchus
C. The foreign body is best removed using rigid bronchoscope
D. X-ray chest will show expiratory emphysema on involved side
E. Cheat X-ray will show Mediastinal shift to involved side
482. 34-A thirty year old lady has five children. She is complaining of
hoarseness for last one month which is not improving. She never had
any surgery .Her probable diagnosis is?
A. Carcinoma of larynx
B. Vocal cord nodules
C. Reinke
,s edema
D. Hypothyroidism
E. Recurrent laryngeal nerve palsy.
35. A six months old baby has strider which gets worse on crying and
improves on lying in prone position .He is suffering from?
A. Laryngeal web
B. Subglottic stenosis
C. Laryngomalscia
D. Congenital paralysis of vocal cord
E. Vocal cord polyp
36. Absolute indication for tonsillectomy is?
A. Chronic tonsillitis
B. Rheumatic fever
C. Diphtheria carrier
D. Obstructive sleep apnea
E. Glassopharyngeal neurectomy
37. Which of the following statements is not rue regarding
oropharyngeal herpes zoster?
A. Vesicles appear in the distribution of ninth and tenth cranial nerves
B. Vesicles are unilateral
C. Pharyngeal condition may be associated with geniculate herpes
D. Healing occurs leaving scarring
E. Acyclovir when used early is helpful.
38. Which of the following is not a clinical feature of adenoid
hypertrophy?
A. Nasal obstruction
B. Snoring
C. Eustachian tube obstruction
D. Nocturnal cough
E. Chronic suppurative otitis media
483. 39-Which of the following is not true regarding acute reteropharyngeal
abscess?
A. It is limited to one side of the midline
B. It forms due to suppuration in reteropharyngeal lymph node
C. Produces difficulty in breathing and suckling
D. It is common in children
E. Incision and drainage is done from outside trough carotid sheath.
40. Surgical emphysema after trachestomy is corrected by:
a)taking more stitches of the wound
b)cold compresses
c)widening of the wound by removal of some stitches
d)antihistaminic intake.
41. Saddle nose may be due to the following except:
a)overresection of septal cartilage
b)nasal trauma
c)septal abcess
d)rhinosceleroma
42. The most serious complication after tonsillectomy:
a)respiratory obstruction
b)reactionary haemorrhage
c)incomplete removal
d)infection.
43. Otoscopic manifestation of chronic secretory otitis media may
include the following except:
a)perforation at pars flaceida
b)transverse handle of malleous
c)absent cone of light
d)air bubbles behind the tympanic membrane.
44. A newly born infant with respiratory distress & difficult feeding is
more likely to be due to:
a)laryngeal web at the anterior half of vocal cords
b)bilateral posterior choanal atresia
c)congenital subglottic stenosis
d)congenital meatal atresia
484. 45-The most common cause of oroantral fistula is:
a)acute sinusitis
b)car accident
c)dental extraction of upper second premolar tooth
d)radical antrum operation.
46. The most common cause for posterior septal perforation is:
a)TB
b)syphilis
c)leprosy
d)scleroma.
47. Moure's sign is:
a)presence of laryngeal click
b)absence of laryngeal click
c)external neck swelling
d)internal pharyngeal swelling.
48. The following symptoms are true of primary atrophic rhinitis except:
a)bad odour felt by the patient
b) bad odour felt by others
c)epistaxis
d)sense of nasal obstruction
49. Unilateral malignant tumor of the vocal cord with subglottis
ectension & lymph node metastasis is best treated by:
a)unilateral cordectomy
b)total laryngectomy with neck dissection
c)cheomotherapy
d)tracheostomy only.
50. For assessment of the skull base and intracranial extension of
cancer,
which combination of studies is most informative?
A. PET and CT
B. CT and MRI
C. PET and MRI
D. MRI and MR angiography
E. PET/CT and MRI .
485. 51-The MR findings of glomus jugulare paraganglioma include a:
A. T1 and T2 ?salt and pepper? appearance.
B. T1 and T2 ?ground glass? appearance.
C. T1 and T2 high signal for cystic necrosis.
D. T1 and T2 high signal within the petromastoid air cells.
52. A jugular foramen schwannoma
A. Has no ?pepper? flow voids, even if it is a large tumor.
B. Has characteristic flow voids.
C. Is a benign tumor of sympathetic nerves in the jugular foramen.
D. Has a high T1 signal.
53-The classic imaging finding in clival meningioma is
A. ?Dural tails.?
B. Flow voids.
C. Central calcification.
D. Peripheral necrosis.
54. The classic CT appearance of fibrous dysplasia of the skull base
includes
A. Pagetoid mixed pattern.
B. Sclerotic pattern.
C. Cystic pattern.
D. All of the above.
55. Langerhans histiocytosis of the skull base has also been known as
A. Hand-Schüller-Christian disease.
B. O?Neill-Morris-Wreesman syndrome.
C. Schüller Siwe syndrome.
D. Delkargios syndrome .
56. All of the following structures extend through the foramen ovale
except the?
A. Mandibular nerve.
B. Cranial nerve (CN) V3.
C. Meningeal branch of the mandibular nerve.
D. Accessory meningeal artery.
486. 57-All of the following structures extend through the jugular foramen
except the
A. Inferior petrosal sinus.
B. Glossopharyngeal nerve.
C. Middle meningeal artery.
D. Sigmoid sinus.
58. All of the following structures extend through the foramen magnum
except the
A. Meninges.
B. Vertebral arteries.
C. Posterior meningeal artery.
D. Medulla oblongata.
59. The agger nasi air cell is
A. The largest and most consistent ethmoid air cell.
B. Positioned anterior to the insertion of the inferior turbinate.
C. Positioned anterior to the insertion of the middle turbinate.
D. Consistently pneumatized.
60. The uncinate process may be attached to any of the following
except the
A. Skull base.
B. Lamina papyracea.
C. Nasal septum.
D. Middle turbinate.
61. On a coronal CT scan, the ethmoid roof is noted tobe 7 mm above
the cribriform plate, which makes this a Keros type
A. I.
B. II.
C. III.
D. IV.
487. 62-The sphenoid sinus ostium is generally located
A. Superior to the superior turbinate 1 to 1.5 cm above the roof of the
choana.
B. Inferior to the superior turbinate 1 to 1.5 cm above the roof of the
choana.
C. Medial to the superior turbinate 1 to 1.5 cm above the roof of the
choana.
D. Lateral to the superior turbinate 1 to 1.5 cm above the roof of the
choana.
63. Mandibular swing should not be performed alone when
A. The tumor is lateral to the IC
A. B. The tumor extends beyond the floor of the middle cranial fossa.
C. The tumor involves intracranial structures.
D. Any of the above is true.
64. Which of the following is not a risk factor for oral cancer?
A. Tobacco
B. Alcohol
C. Fanconi anemia
D. Herpes simplex virus
E. Human immunodeficiency virus (HIV) infection.
65. Postoperative chemoradiation therapy is indicated in patients with
A. Positive neck nodes.
B. Vascular invasion.
C. Perineural invasion.
D. T4 tumors.
E. Extracapsular lymph node spreading.
66. In nasopharyngeal cancer the most common site of metastases is
the
A. Bone.
B. Liver.
C. Lung.
D. Brain.
488. 67-NPC is highly sensitive to
A. Doxorubicin (Adriamycin).
B. Methotrexate.
C. Cyclophosphamide.
D. Platinum-based regimens.
68. The preferred imaging study for evaluating the intracranial or
perineural extent of nasopharyngeal tumors is.
A. CT.
B. Enhanced magnetic resonance imaging (MRI).
C. Enhanced CT.
D. Ultrasonography.
69. The parapharyngeal space extends from the skull base to the
A. Hyoid.
B. Level of the epiglottis.
C. Level of the angle of the mandible.
D. Level of the postcricoid region.
70. The masticator space includes the following muscles, except the
A. Medial pterygoid.
B. Lateral pterygoid.
C. Posterior digastric.
D. Temporalis.
71. A nasopharyngeal primary SCC with extension to the oropharynx
without parapharyngeal space extension is stage
A. T1.
B. T2.
C. T3.
D. T4.
72. Esophageal webs are associated with the following, except
A. Plummer-Vinson syndrome.
B. Celiac disease.
C. Gastroesophageal reflux disease (GER
D. .
D. Bullous pemphigoid .
489. 73-Cancer is limited to the True vocal cord, but paralysis is present.
A. T1a
B. T1b
C. T2
D. T3
E. T4a.
74. Vocal cord fixation is most likely due to invasion of the
A. Conus elasticus.
B. Quadrangular membrane.
C. Paraglottic space.
D. Broyle tendon.
75. Which of the following would not be recommended as part of the
treatment for a T1 piriform sinus hypopharynx tumor?
A. Transoral laser resection of the primary tumor with negative margins
B. Radiation therapy alone
C. Transcervical lateral pharyngectomy
D. Bilateral neck dissection after surgery for the primary tumor
E. Concomitant chemoradiation therapy.
76. A 75-year-old woman presents with an incidentally found T1
hypopharynx tumor of the medial piriform sinus. CT and PET findings
are negative for cervical disease. She undergoes surgical resection of
the primary lesion. Which of the following is the best next step in
treatment of the neck?
A. No neck dissection necessary
B. Ipsilateral neck dissection only
C. Bilateral neck dissection
D. Ipsilateral radiation therapy to the neck
E. Bilateral radiation therapy to the neck.
77. The radiological demarcation between level I and level II in the
neck is:
A. A vertical line through the posterior border of the submandibular
gland.
B. The posterior belly of the digastric muscle.
C. The mylohyoid muscle.
D. A vertical line through the tip of the greater cornu of the hyoid bone.
490. 78-Which of the following is most accurate at staging the neck in
patients with SCC of the larynx who have not received any previous
treatment?
A. Clinical palpation
B. MRI
C. PET-CT
D. US-FN
A.
79. The treatment of choice for lymphadenopathy caused by atypical
mycobacteria in children is
A. Rifampicin-based antimicrobial chemotherapy.
B. Sulfa-based antimicrobial chemotherapy.
C. Complete surgical excision without antimicrobials.
D. Surgical curettage and debulking followed by antimicrobial therapy.
80. Sacrifice of which of the following structures constitutes an
extended MRND?
A. Accessory nerve, hypoglossal nerve, internal jugular vein
B. Sternomastoid muscle, omohyoid muscle, and internal jugular vein
C. Accessory nerve, sternomastoid muscle, and internal jugular vein
D. Accessory nerve, internal jugular vein, and submandibular gland.
81. Which of the following statements regarding second arch branchial
cysts is true?
A. The peak age of presentation is in the teenage years.
B. Most cases will have a rudimentary tract extending deeply to the
tonsillar fossa.
C. Approximately 10% to 30% are bilateral.
D. These cysts may become inflamed in patients with acute tonsillitis
because of lymphoid tissue in the wall.
82. In patients with differentiated thyroid cancermetastatic to the lateral
side of the neck, which preoperative investigation should be considered
routine?
A. Ultrasound examination
B. PET scan
C. Contrast-enhanced CT of the neck
D. Diagnostic radioiodine scan
491. 83-Which of the following side effects is associated with the
administration of radioactive iodine?
A. Dysphagia
B. Xerostomia
C. Secondary malignancy
D. All of the above.
84. What is the rate of incidental malignancy in multinodular goiters?
A. Less than 5%
B. 5% to 10%
C. 10% to 30%
D. 30% to 50%.
85. Which of the following areas is a common site for nodal metastases
from thyroid cancers?
A. The perifacial lymph nodes
B. Level IV posterior to the great vessels
C. Level II superior to the accessory nerve
D. The parapharyngeal space
86. Children who are seen with papillary thyroid cancer are at high risk
of
A. Locally advanced primary tumors.
B. Bony metastases.
C. Pulmonary metastases.
D. Local recurrence.
87. The most important differential diagnosis in patients with suspected
anaplastic thyroid cancer is
A. Poorly differentiated thyroid cancer.
B. Lymphoma.
C. Sarcoma.
D. Metastatic melanoma.
88. The next step in managing a patient with normal clinical
examination results and a rising thyroglobulin level after surgery for
thyroid cancer is.
A. Elective neck dissection.
B. Ultrasound imaging.
C. Therapeutic radioiodine.
D. Tyrosine kinase inhibitor therapy.
.
492. 89-The most reliable landmark for the facial nerve in parotid surgery is
the
A. Tragal pointer.
B. Posterior belly of the digastric.
C. Tympanomastoid suture line.
D. Bony cartilaginous ear canal.
90. Characteristic MRI findings for pleomorphic adenoma include
A. Hyperintensity on T1.
B. Hyperintensity on T2.
C. Enhancement with gadolinium.
D. A and
B. E. B and
C.
91. Which of the following statements about recurrent minor salivary
gland cancer is correct?
A. Local recurrences require chemoradiation therapy.
B. Local recurrences indicate a palliative course.
C. Local recurrences do not necessarily portend a decreased chance of
survival.
D. Surgical intervention is not a therapeutic option for local recurrences
92. Paragangliomas often have demonstrable feeder vessels that most
commonly arise from the
A. External carotid artery.
B. Internal carotid artery.
C. Ascending pharyngeal artery.
D. Lingual artery.
E. Superior thyroid artery.
93. The parapharyngeal space is bounded by the following structures,
except the
A. Base of the skull.
B. Lateral pharyngeal wall.
C. Masseter muscle.
D. Pterygoid muscles.
E. Prevertebral muscles..
493. 94-A 35-year-old man presents with a 3-cm schwannoma of the
cervical plexus. The best next step is
A. Observation with repeated imaging in 6 months.
B. Radiation therapy.
C. Chemotherapy.
D. Early surgical intervention.
95. The differential diagnosis of a 3-cm mass on the right side of the
neck located in the carotid space should include all of the following,
except a
A. Glomus vagale tumor.
B. Carotid body tumor.
C. Schwannoma of the vagus nerve.
D. Pleomorphic adenoma of the deep lobe of the parotid gland .
96. Which vein is not a reliable recipient for venous drainage of a free
flap?
A. External jugular vein
B. Internal jugular vein
C. Facial vein
D. Anterior jugular vein
E. Middle thyroid vein.
97. Which nerve is used in the sensate radial forearm free flap?
A. The lateral antecubital cutaneous nerve is used.
B. The sensory branch of the radial nerve is used.
C. The radial forearm flap cannot be raised as a sensate flap.
D. The medial antecubital cutaneous nerve is used.
E. The posterior cutaneous nerve of the forearm is used.
98. Which is true regarding the deltopectoral flap?
A. The donor site can be closed primarily.
B. It is the preferred choice for pharyngeal reconstruction.
C. The blood supply is from the axillary artery.
D. It should be raised as a musculocutaneous flap to preserve the blood
supply.
E. The distal end of the flap has the most tenuous blood supply.
494. 99-The blood supply to the skin paddle of the fibula osteocutaneous
flap
A. Is highly unreliable.
B. Is always anterior to the fibula.
C. Is always in the posterior intermuscular septum.
D. Is usually located in the proximal one third of the bone.
E. Can be better protected with harvesting of acuff of soleus or flexor
hallucis longus muscle.
100. Which is true regarding the pectoralis major muscle?
A. The blood supply is from the pectoral branch of the thoracoacromial
artery.
B. The nerve supply is from the long thoracic nerve.
C. The main blood supply to the muscle is from the internal mammary
artery.
D. The lateral thoracic artery should be preserved.
E. The nerve supply is from the axillary nerve.
101. Which one is not a pivotal flap?
A. Bilobed flap
B. Rhombic flap
C. V-Y flap
D. Z-plasty
E. Paramedian forehead flap.
102. Which one of these defects is better reconstructed with a free
flap?
A. A glossectomy defect less than 30%
B. A lateral marginal mandibulectomy defect in a nonirradiated patient
with more than 1 cm of thickness of remaining mandible
C. A mucosal defect of the floor of the mouth
D. A mucoperiosteal defect of the hard palate
E. A hemiglossectomy defect
495. 103-Which vessel is not a first choice option in microvascular
anastomosis in the neck?
A. Superior thyroid artery
B. Facial artery
C. Lingual artery
D. Transverse cervical artery
E. Occipital artery.
104. Which tumor is most likely to have retropharyngeal lymph node
metastasis?
A. Nasopharynx
B. Oral cavity
C. Glottic larynx
D. Parotid .
105. Which of the following is the most common late sequela after
radiation therapy to the head and neck?
A. Osteoradionecrosis
B. Feeding tube dependence
C. Hypothyroidism
D. Brachial plexopathy.
106. After radiation therapy, which of the following is not a suspicious
feature of a persistent metastatic lymph node?
A. Size greater than 1.5 cm
B. Focal enhancement on CT
C. Well-circumscribed node
D. Calcification.
107. The presence of peripheral cysts around the intracranial
component of a sinonasal lesion is generally associated with which of
the following tumors?
A. Sinonasal undifferentiated carcinoma
B. Melanoma
C. Esthesioneuroblastoma
D. Squamous cell carcinoma
E. Adenoid cystic carcinoma.
496. 108-Which of the following tumors is not commonly associated with
lateral retropharyngeal lymphadenopathy?
A. Nasopharyngeal carcinoma
B. Paranasal sinus carcinoma
C. Tonsil carcinoma
D. Papillary carcinoma of the thyroid
E. Pyriform sinus carcinoma.
109. Hypopharyngeal cancers can cause paralysis of the ipsilateral vocal
cord by all of the following mechanisms except
A. Direct invasion of the paraglottic space.
B. Direct invasion of the cricoarytenoid joint.
C. Perineural involvement of the recurrent laryngeal nerve.
D. Direct invasion of the recurrent laryngeal nerve.
E. Involvement of the recurrent laryngeal nerve by paratracheal
lymphadenopathy.
110. Insulin Diabetic Patient undergoing total thyroidectomy, should
a. Be admitted 1 hour before surgery
b. Receive pre operative insulin pre-
c. Receive intravenous normal saline preoperatively
d. Receive a glucose, insulin and potassium infusion preoperatively
e. Should receive intravenous insulin intraoperatively.
111. Total serum thyroxine is reduced by
a. Oral contraceptive pills
b. Pregnancy
c. Nephrotic Syndrome
d. Propanolol.
112. Facial Nerve Is Not Fully Developed Untill A Childe Is:
A. 1 Year
B. 2 Year
C. 4 Year
D. 6 Year
E. 8 Year.
497. 113-The Differential Diagnosis Of Acute Facial Nerve Parlysis , Should
Not Include:
A. Sarcoidoisis
B. Herbes Zoster
C. Kawasaki?s Disease
D. Lyme Disease
E. Shwanoma.
114. The most common source of infection of the lateral pharyngeal
space is from the
A. Floar of the mouth
B. Neck
C. Mastoid
D. Soft palate
E. Tonsils.
115. lateral pharyngeal space is devided into two compartments by the
A. 11th cranial nerve
B. Carotid sheath
C. Styloid process
D. Carotid artery
E. Jugular artery .
116. parotid gland is separated from submandibular gland by
A. Mylohyoid
B. Stylohyoid
C. Stylomandibular ligament
D. Posterior belly of digastric.M
E. Anterior belly of digastric,m.
117. A congenital choanal atresia is:
A. Most commonly a membranosious closure
B. Mostly aunilateral
C. Commoner in female
D. Of early presentation in unlilateral cases.
E. Not a life threatening condition jn bilateral cases
498. 118-The interaural difference in wave V of ABR must not exceed :
A. 0.2 msec
B. 1 msec
C. 0.1 msec
D. 2.4 msec
119. Which of the following is MOST specific for otosclerosis:
A. Hyperacusis Willichii
B. Normal tymponometry with lost reflexes.
C. Carhart's notch.
D. Schwart's sign.
E. None of the above
120. 3 years old child with sinusitis , you expect affected sinuses are :
A. Frontal and maxillary.
B. Frontal and ethmoidal.
C. Sphenoidal and maxillary
D. Maxillary and ethmoidal.
121. Patient with right complete facial paralysis is working in a noisy for
20. years,you can expect:
A. Right ear hearing loss is less at 4kHz.
B. Left ear hearing is worse than right.
C. Both ears got equal hearing.
D. Right ear hearing loss is more at 4kHz .
122. Cochlear microphonics originate from:
A. Cochlear nerve.
B. Basilar membrane.
C. Outer hair cells.
D. Inner hair cells.
123. Compoud action potential originates from:
A. Auditory nerve.
B. Basilar membrane.
C. Outer hair cells.
D. Inner hair cells..
499. 124-The endocochlear potential originates from:
A. Basilar membrane.
B. Outer hair cells.
C. Stria vascularis.
D. Inner hair cells .
125. The 4th branchial arch nerve develops into:
A. The glossopharyngeal nerve.
B. Vagus nerve.
C. Mandibular nerve.
D. Superior laryngeal nerve.
126. The most common complication of FESS:
A. Bleeding.
B. Cranial complictions.
C. Anosmia.
D. Synaechia..
127. The MOST common route of spread of supraglottic carcinoma :
A. Lymphatic.
B. Blood spread.
C. Anteriroly through fenestra of cartilage to pre-epiglottic space.
D. Inferiorly to the lung.
128. Early signs of respiratory failure:
A. Cyanosis.
B. Tachypnea.
C. Subcostal retraction.
D. Biphasic stridor.
E. Irritability and gasping.
129. Exostosis is characterised by all except:
A. Multiple.
B. Bilateral.
C. Peduculated.
D. Usually in cold water divers.
E. Lamellated bone .
500. 130-In a patient with right paralytic labyrinth, which of the following is
correct:
A. Right beating nystagmus.
B. Left beating nystagmus.
C. Vertical nystagmus.
D. Total hearing loss.
131. In acoustic neuroma, you can find:
A. High frequency SNHL.
B. Low frequency SNHL.
C. Flat audiogram.
D. Notch at 2 KHZ.
132. In modified radical neck dissection type I , which of the following
will be preserved:
A. Thoracic duct.
B. Spinal accesory nerve.
C. Internal jugular vein.
D. Sternomastoid muscle.
E. 2 & 3.
133. Trotter' triad consisit of all of the following except:
A. Bilateral conductive hearing loss.
B. Otalgia and facial pain.
C. Unilateral immobile soft palate.
D. Unilateral ear effusion.
134. The nasopalatine nerve supplies sensation to the mucosa of the?
A. Anterior premaxillary palate.
B. Soft palate.
C. Lateral nasal passage.
D. Anterior cheek.
135. The most commonly injured branch of the facial nerve in
rhytidectomy is the?
A. Temporal.
B. Marginal mandibular.
C. Buccal.
D. Cervical.
E. Zygomatic .
501. 136-Infection spread from the maxillary sinus intracranially most
likely travels through the ?
A. Angular veins.
B. Ophthalmic artery.
C. Superior labial veins.
D. Pterygoid plexus .
137. Which virus is especially associated with unilateral sensorineural
hearing loss?
A. cytomegalovirus
B. rubella
C. rubeola
D. adenovirus
E. mumps.
138. Susceptibility to noise-induced hearing loss may be potentiated by
A. genetic predisposition.
B. smoking.
C. the gender of the subject.
D. excessive alcohol intake.
E. heart disease.
139. Which is the most common traumatic ossicular abnormality?
A. necrosis of the long process of the incus
B. incudostapedial joint disarticulation
C. fracture of the stapes suprastructure
D. fixation of the incus
E. dislocation of the stapes footplate.
140. Which of the following is untrue about Waardenburg?s syndrome
(WS)?
A. It may be associated with Hirschsprung?s disease.
B. Dystopia canthorum occurs in WS1 and WS3.
C. Hearing loss is typically mixed in nature.
D. Profound hearing loss is common.
E. It may be associated with heterochromia iridis.
502. 141-Branchio-oto-renal syndrome may be associated with the
following, except
A. dilated vestibular aqueduct.
B. autosomal recessive inheritance.
C. mutations in EYA1.
D. renal anomalies.
E. marked phenotypic variability.
142. Which of the following statements is most accurate about
congenital hearing loss?
A. The incidence is decreasing.
B. Fifty percent or more is inherited.
C. It is always profound.
D. It may be late in onset.
E. It is nearly always sensorineural in nature.
143. The most reliable means of improving hearing in the patient with
otosclerosis is a
A. hearing aid.
B. fenestration operation of Lempert.
C. stapedectomy.
D. stapedotomy
E. partial stapedectomy.
144. The principal objective in stapes surgery is to
A. protect the inner ear from intense sound.
B. increase round window membrane mobility.
C. restore ossicular continuity.
D. restore sound pressure transmission to the inner ear.
E. increase fluid movement in the inner ear.
145. Which of the following does not describe Carhart?s notch?
A. ranges in magnitude up to 15 dB
B. is centered at 2 kHz
C. is an apparent sensorineural hearing loss
D. usually disappears after stapes surgery
E. occurs without a conductive hearing loss.
503. 146-The most frequent site of first occurrence of otosclerosis is in the
A. oval window rim.
B. fossula post fenestram.
C. round window niche.
D. fissula ante fenestram.
147. Computed tomography scanning is superior to magnetic resonance
imaging in establishing which one of the following diagnoses?
A. Bezold?s abscess
B. coalescent mastoiditis
C. epidural abscess
D. cerebellar abscess
E. subdural empyema.
148. Critical steps in constructing a canal wall down mastoid cavity
include all of the following except
A. removal of the mastoid tip.
B. obliteration of the mastoid with soft tissue.
C. wide saucerizarion of the cavity.
D. always drilling parallel to the facial nerve and using copious irrigation
to identifythe sheath before exposing it.
E. removal of conchal cartilage when performing the meatoplasty.
149. The most critical factor in avoiding injury to the facial nerve during
mastoid operations is
A. use of a facial nerve monitor.
B. a preoperative CT scan.
C. drilling parallel to the nerve.
D. using a diamond bur.
E. exposing the sheath of the nerve by removing the overlying bone.
150. Therapy that has been proven to help in otitis media (OM) includes
all of the following except
A. antibiotics.
B. antihistamines.
C. corticosteroids.
D. tympanostomy tube placement.
E. analgesics.
504. 151-Advanced malignancies of the temporal bone are best treated with
a. Surgery only
b. Radiation only
c. Surgery and radiation
d. Chemotherapy
152. What is the most common cause of inoperability in congenital
aural atresia?
a. Absence of the oval window
b. A facial nerve overlying the oval window
c. Poor mastoid pneumatization
153. One of these complications is not seen with underlay
tympanoplasty grafting:
a. Reperforation
b. Blunting
c. Infection
d. Myringitis of the graft.
d. Unilateral atresia.
154. Which one of following statements is correct?
a. Blunting is a complication of underlay tympanoplasty.
b. The rate of take for overlay tympanoplasty is greater than for
underlay tympanoplasty.
c. Lateralization is a complication of underlay tympanoplasty.
d. Myringitis is not seen after underlay tympanoplasty.
155. Cartilage should be considered as a graft material in the following
situations:
a. The atelectatic ear
b. Cholesteatoma
c. A perforation anterior to the annulus
d. A draining perforation
e. All of the above
156. Contraindications to canal wall reconstruction
tympanomastoidectomy include
a. Mastoid cholesteatosis
b. Sinus tympani involvement
c. Facial paralysis
d. Tegmen defect with meningoencephalic herniation
505. 157-The most common intraoperative finding associated with revision
stapedectomy is
a. Perilymph fistula
b. Prolapsed facial nerve
c. Displaced prosthesis
d. Fibrosis of oval window tissue.
158. Presence of acoustic reflexes on a preoperative audiogram should
prompt the operating surgeon to
a. Cancel the surgery.
b. Order a CT scan.
c. Consider superior semicircular canal dehiscence as a cause of the
conductive hearing impairment.
d. Do all of the above.
159. What segment of the facial nerve is most commonly damaged as a
result of temporal bone trauma?
a. Intracranial
b. Meatal
c. Labyrinthine/perigeniculate
d. Tympanic
160. Which of the following is considered to be an absolute
contraindication to cochlear implantation?
a. Duration of deafness greater than 30 years
b. Auditory neuropathy
c. Enlarged vestibular aqueduct
d. Michel aplasia
e. All of the above
161. An absolute contraindication to middle fossa vestibular nerve
section is
a. An only hearing ear
b. Bilateral Ménière?s disease
c. Incapacitating vertigo
d. Diuretic allergy
e. Age less than 60 years
506. 162-The most common form of BPPV results from
a. Anterior canal canalithiasis
b. Lateral canal cupulolithiasis
c. Lateral canal canalithiasis
d. Posterior canal cupulolithiasis
163. What is the most common serious complication of superior canal
dehiscence (SC
D. plugging surgery in the initial 24 hours after surgery?
a. Aphasia
b. Hematoma
c. Seizure
d. Meningitis
e. Posterior canal canalithiasis
164. Gradenigo?s syndrome usually includes all of the following except
a. Retro-orbital pain
b. Fourth cranial nerve palsy
c. Otorrhea
d. Hearing loss
e. Sixth nerve palsy.
165. Which of the following drugs is most likely to cause permanent
hearing loss?
A. erythromycin
B. quinine
C. cisplatin
D. penicillin G
166. A 30 years old patient with recurrent attacks of vertigo, hearing
loss and tinnitus associated with nausea and vomiting has
a- benign paroxysmal positional vertigo
b-vestibular neuronitis
c-Meniere?s disease
167. The inner ear is ebmbryologically derived from
A. ectoderm
B. entoderm
C. mesoderm
D. all of the above
507. 168-All of the following are examples of absorbable suture, except
which of the following?
A. Polyglactin
B . Poliglecaprone 2 5
c. Polypropylene
D . Polydioxanone
d-acoustic neuroma.
169. In a diagnostic workup of headache, further radiologic evaluation
with MRI or cr scan is indicated in which of the following
circumstances?
A. Pulsatile headaches
B. Headache awakening one from sleep
C. Onset of headache in childhood
D. Unilateral headache
170. Which drug is most likely to be effective for the treatment of
migraine-associated vertigo?
A. Sumatriptan
B . Meclizine
c. Nortriptyline
D . Diazepam
171. Which of the following descriptions is characteristic of carcinoma in
situ?
A. Pushing borders with mild atypia along the basal layer
B. Mild atypia, but with violation of the underlying basement membrane
C. Full-thickness cellular atypia with an intact basement membrane
D. Moderate atypia extending into the upper third of the mucosa .
172. Robotic surgery benefits from:
A. 360° wristed motion
B . Scaled motion with tremor suppression
C. Binocular magnification
D. Both B and C
E. All of the above
508. 173-A cerebrospinal fluid leak during ethmoidectomy is most likely to
occur at:
A. The junction of fovea ethmoidalis and lamina papyracea
B. The medial fovea ethmoidalis
C. The insertion of the uncinate process
D. The planum sphenoidale
174. To best determine whether the lamina papyracea is intact, which
test or procedure should be used?
A. Intraoperative CT scan
B. Fat float test
C. Bulb press test
D. Endoscopic dissection and search for orbital fat
175. Which of these is the most commonly identified organism in
subperiosteal orbital abscess due to sinusitis?
A. Pseudomonas aeruginosa
B. Haemophilus influenzae
C. Streptococcus pneumoniae
D. Streptococcus viridans .
176. What is the initial radiographic study of choice in a patient with
suspected cerebrospinal fluid (CSF) leak from accidental trauma?
A. High-resolution CT scan
B. MR imaging
c. cr cisternogram
D. Radionuclide cisternogram
E. MR cisternogram
177. Which of the following is not considered part of the nasal valve?
A. Head of the inferior turbinate
B. Bony piriform aperture
C. Nasal floor
D . Membranous septum
509. 178-Which of the following most commonly causes nasal valve
obstruction?
A. Previous rhinoplasty
B. Turbinate hypertrophy
C. Nasal polyposis
D. Congenital
179. Which of the following glands produce the majority of the
unstimulated saliva?
A. Parotid glands
B. Minor salivary glands
C. Sublingual glands
D. Submandibular glands .
180. Based on the multicellular theory of tumorigenesis,
mucoepidermoid carcinoma arises from:
A. Excretory duct cells
B. Acinar cells
C. Striated duct cells
D. Intercalated duct cells
181. Workup for a new diagnosis of vocal fold paralysis does not
routinely include:
A. Cf scan
B . Laryngeal electromyography (LEMG)
C. Swallowing assessment
D. Serology
182. What is the first-line treatment for recurrent respiratory
papillomatosis?
A. Surgical removal
B. Inhaled cidofovir
C. High-dose oral steroid burst
D. Azathioprine
E. Radiation therapy
510. 183-Stroboscopy allows assessment of the following except:
A. Vocal fold closure
B. Vocal fold mobility
C. Mucosal pliability
D . Vocal fold level during phonation
E. Vocal fold sub-epithelial pathology .
184. Continuous positive airway pressure (CPAP) is effective for
obstructive sleep apnea because it:
A. Supports the soft palate
B. Compresses the tongue base
C. Stimulates activity of dilating muscles
D. Prevents pharyngeal collapse during expiration
E. Forces the pharynx open after obstruction
185. Temporary vocal fold augmentation materials include:
A. Silas tic
B. Hyaluronic acid
c. Titanium
D . Gore-Tex
E. Teflon
186. Which of the following is the most common cause of iatrogenic
bilateral vocal fold paralysis?
A. Cervical spine surgery
B . Lung surgery
c. Thyroid surgery
D . Carotid surgery
187. Where are septal fractures most commonly seen?
A. Above the interface with the maxillary crest
B. At the caudal septum
C. At the junction of the cartilage with the perpendicular plate of the
ethmoid bone
D. Right where the cartilage interfaces with the maxillary crest
E. Mediastinoscopy
511. 188-Which of the following is one of the most important factors in
successful frontal sinus obliteration?
A. Choosing the correct obliteration material
B. Complete removal of all sinus mucosa
C. Choosing the appropriate plate size for posterior table reconstruction
D . Use of atraumatic technique for elevation of the pericranial flap
189. Which of the following nasal bone fractures are most likely to be
associated with other facial fractures?
A. Displaced nasal fractures
B. Fractures of the caudal aspect of the nasal bones
C. Fractures of the cephalic end of the nasal bones
190. Which is not a vertical buttress of the face?
A. Zygomaticomaxillary
B. Zygomatic arch
C. Nasomaxillary
D. Pterygomaxillary
D. Fractures of the nasal sidewall
191. Robin sequence is characterized by:
A. Micrognathia, cleft palate, and glossoptosis
B . Micrognathia, cleft palate, and airway obstruction
C. Cleft palate, airway obstruction, and glossoptosis
D. Micrognathia, glossoptosis, and airway obstruction.
192. Which of the following is the second most commonly occurring
type of tracheoesophageal fistula (TEF) ?
A. Proximal TEF with distal esophageal atresia (E
A. B. Proximal EA with distal TEF
C. H-type TEF
D . Proximal and distal TEF with EA
193. Which of these is the most likely diagnosis in a neonate with
expiratory stridor and a brassy cough?
A. Congenital subglottic stenosis
B. Tracheomalacia
C. Robin sequence
D . Bilateral vocal fold paralysis
512. 194-. Imaging for pediatric rhinosinusitis is most accurately
accomplished with:
A. Plain films
B. CT scans
C. MRI
D . Maxillary sinus transillumination
195. A child with bilateral second branchial cleft anomalies {BC
A. may
have all of the following except:
A. Bilateral preauricular pits
B . Sensorineural hearing loss
C. Renal disease
D. Microtia
E. Autosomal recessive inheritance.
196. Recurrent respiratory papillomatosis (RRP) demonstrates a
predilection for which of the following sites?
A. Junction of ciliated and squamous epithelium
B. The undersurface of the vocal folds
C. The laryngeal ventricle
D. All of the above
197. Which of the following is currently the most common indication for
tracheotomy in the pediatric population?
A. Providing access for pulmonary toilet
B. Airway obstruction from subglottic stenosis
C. Airway obstruction from craniofacial abnormalities
D. Respiratory failure and ventilator dependence
198. All of the following are appropriate associations on radiologic
evaluations except:
A. Steeple sign-laryngotracheitis
B. Thumbprint sign-epiglottitis
C. Air trapping on chest radiograph-vocal cord immobility
D. Thickened soft tissue overlying
C. spine-retropharyngeal
abscess/phlegmon
513. 199-The most common presenting symptoms in children with recurrent
respiratory papillomatosis (RRP) are:
A. Dysphagia and poor weight gain
B. Dyspnea with exertion
C. Progressive hoarseness and stridor
D. Odynophagia
200. Which laryngeal site is the most at risk of developing
radionecrosis?
A. Cricoid
B . Thyroid lamina
c. Arytenoid
D . Epiglottis
201. Which of the following salivary cancers is most associated with
pain and perineural spread of tumor?
A. Acinic cell cancer
B. Adenoid cystic cancer
C. High-grade mucoepidermoid cancer
D. Squamous cell cancer
202. Fibrous dysplasia is commonly treated with:
A. Observation
B . Recontouring and esthetic surgery
C. Wide local excision
D. Chemotherapy.
203. The most common acute toxicity observed after oral radiation
therapy are:
A. Mucositis, osteonecrosis, and xerostomia
B . Mucositis, carotid endarteritis, and xerostomia
C. Mucositis, dysphagia, and xerostomia
D. Mucositis, fibrosis, and xerostomia
204. The commonest presenting symptom of nasopharyngeal (NP)
carcinoma is
A. Epistaxis
B. Hearing loss due to serous otitis media
C. Enlarged cervical lymph node
D. Nasal obstruction
514. 205-Which of these is the most common presenting symptom( s) for
tracheal adenoid cystic carcinoma?
A. Hemoptysis
B . Dyspnea
c. Fever
D . Dysphagia
206. What is the only absolute contraindication to primary
tracheoesophageal puncture?
A. Obesity
B. Prior radiation
C. Partial pharyngectomy
D. Separation of trachea and esophageal party wall
E. Neck mass
E. Abducent nerve palsy.
207. Accurate diagnosis of obstructive sleep apnea {OS
A. can best be
made by:
A. A careful history with a complete review of systems
B. Bed partner history
C. Physical examination of the upper airway
D . A home sleep study or an in-lab polysomnogram
208. Clinical outcomes in the assessment of obstructive sleep apnea
(OS
A. treatment include:
A. Apnea-hypopnea index
B. Daytime sleepiness
C. Airway collapsibility
D. Oxygen desaturation
209. What is considered the gold standard for treatment of obstructive
sleep apnea in adults?
A. Nasal septoplasty
B. Positive airway pressure therapy
C. Uvulopalatopharyngoplasty
D. Tongue base reduction therapy
515. 210-Most cases of sudden sensorineural hearing loss (SSNHL) are
caused by:
A. Temporal bone trauma
B. Genetic predisposition
C. Neurologic disease
D. No identifiable source \ediopathic .
211. Spontaneous hearing recovery after a sudden sensorineural
hearing loss, if it is to occur, occurs within what time frame?
A. 3 days
B. 2 weeks
C. 2 months
D. 6 months
212. Which of the following indications in congenital aural atresia is the
most important determinant of surgical candidacy by CT scan?
A. Degree of mastoid pneumatization
B . Size and position of ossicles
C. Thickness of atretic bone
D. Size of middle ear
213. Which of the following is true regarding the presence of
semicircular canal fistula from erosion by cholesteatoma?
A. Most patients complain of vertigo .
B. The maj ority involve the posterior semicircular canal .
C. Removal of the matrix from a fistula inevitably results in an
anacoustic ear.
D. It is best managed with topical neomycin.
E. In an extensive fistula with a contacted mastoid, a canal-wall-down (
CW
D. procedure is best .
214. Which of the following define the superior boundary of the sinus
tympani:
A. Subiculum.
B. Fossula of the cochlear window.
C. Meatal fossa.
D. Ponticulus.
E. Fossaovalis..
516. 215-The most common congenital ossicular anomalies is :
A. Stapes anomalies.
B. Malleus anomalies.
C. Incudo-stapedeal joint anomalies.
D. Incudo-malleal joint anomalies.
E. Incus anomalies.
216. Acute sppurative otitis media associated to facial palsy is adequatly
treated with:
A. Myringotomy and systemic A
B. B- Cortical mastoidectomy with systemic A
B. C-Cortical mastiodectomy with facial n. decompression.
D. Myringotomy with facial n.decompression.
E. Systemic AB .
217. The best line of treatment of uncontroled unilateral Meniere?s
disease with sever Vertigo and profound SNHL is ?
A. intrtympanic injection of corticisteroids.
B. intratympanic injection of gentamycin.
C. Labrinthectomy.
D. Vestibular nerve neuroectomy.
E. Ultrasound destruction of semicircular canals.
218. CSF is produced by a process of ultrafiltration mainly from the
following cells:
A. Ependymal cells.
B. Epithelial cells of choroid plexus.
C. Glial cells.
D. Intimal cells of I
C. blood vessels.
E. Ventricular supporting cells.
219-The most common cause of unilateral proptosis in adults is:
A. orbital pseudotumour
B. Graves ophthalmopathy
C. paranasal isnus tumours
D. lymphomas
E. meningioma
.
517. Except
In salivary gland tumours all the following are true
220.
A. The commonest parotid tumour is pleomorphic adenoma.
B. The adenolymphoma (Warthin?s tumour) is commoner in young
women, is painful and grows rapidly.
C. A submandibular tumour is more likely to be malignant than a parotid
tumour.
D. In mucoepidermoid carcinoma, recurrence rates and survival
correlate
with histological grade.
E. Distant metastases after many years are characteristic of
adenoidcystic carcinoma.
221. Malignant changes of long standing recurrent pleomorphic
adenoma of the parotid are suspected:
A. If the recurrence is a single module of tumor.
B. If the recurrence is multiple small modules.
C. When the recurrence is in the deep lobe of parotid.
D. When previous surgery was a simple lumpectomy.
E. If patient had previous surgery plus radiotherapy .
222. The most reliable diagnostic tool for lateral sinus thrombosis is:
A. MRI.
B. MR
A. C- CT scan.
D. Echo of high neck.
E. Lumbar puncture
223. The auriculo temporal nerve
A. supplies the skin of the upper 2/3 of the lateral surface of the auricle
B. is a branch of maxillary nerve
C. supplies the middle ear mucosa through the tympanic plexus
D. non of the above
224. The Eustachian tube is opened by contraction of
A. tensor tympani muscle
B. levator palati muscle
C. tensor palati muscle
D. Salpingopharyngeus muscle
518. 225-The sensory end-organ of the semicircular canal is :
a- the organ of Corti
b- the macula
c- the crista
d- non of the above .
226. All of the following are diagnostic of tympanic membrane
retraction except
a- fore-shortened handle of malleus
b- prominent lateral process of malleus
c- Schwartz sign
d- distorted cone of light
227. Bezold abscess is a collection of pus :
a- above and in front of the auricle
b-behind the auricle
c-in the upper part of the neck deep to the sternomastoid
d- in the peritonsillar space
228. The most common complication of myringotomy operation is
a- injury of facial nerve
b-dislocation of the incus
c-injury of the jagular bulb
d- residual perforation
229. Intermittent fever with rigors and headach in a patient with
cholesteatma may be due to :
a-otogenic meningitis
b-otogenic brain abscess
c- lateral sinus thrombophlebitis
d-extradural abscess
230. A persistent profuse ear discharge after acute otitis media is
a- cholesteatoma
b- secretory otitis media
c-mastoiditis
d- diffuse otitis externa
519. 231-In otitic barotrauma, the following statements are correct except:
a-occurs during airplane ascent
b- occurs during airplane rapid descent
c- can cause rupture of the tympanic membrane
d- occurs during diving
232. The commonest cause of vertigo is
a- meniere?s disease
b-labyrinthitis
c-benign paroxysmal positional vertigo
d-ototoxicity
233. Most cases of extradural abscess of the temporal lobe
a- are asymptomatic and discovered accidentally during mastoidectomy
b- present with persistent ipsilateral temporal headach
c- present with vertigo
d- present with pulsating discharge,hearing loss and tinnitus .
234. The type of hearing loss in otosclerosis may be
a- conductive
b- sensorineural
c-mixed
d-all of the above
235. A 30 years old patient with recurrent attacks of vertigo, hearing
loss and tinnitus associated with nausea and vomiting has
a- benign paroxysmal positional vertigo
b-vestibular neuronitis
c-Meniere?s disease
d-acoustic neuroma
236. In lower motor neurone facial paralysis with intact taste sensation
at the anterior 2/3 of the tongue, the level of the lesion is :
a- in the internal auditory canal
b-in the horizontal tympanic part
c- in the vertical part above the stapes
d- in the stylomastoid foramen
520. 237-By modified radical mastoidectomy we mean
a- removal of mastoid air cells and all middle ear contents
c- removal of diseased mastoid air cells
c- removal of mastoid air cells and all middle ear contents with
preservation of healthy remnants of tympanic membrane and ossicles
d- non of the above .
238. Which of the following is associated with objective tinnitus
a-Menière's disease.
b-Ear wax impaction.
c-Acoustic neuroma.
d-Palatal myoclonus.
e-Middle ear effusion
239. THE SECOND COMMONEST CPA MASS LESION IS:
A. GLIOMA
B. ANEURYSM
C. MENINGIOMA
D. EPIDERMOID
240. External auditory canal receives blood supply from all the arteries
except:
A. Posterior auricular
B. Superficial temporal
C. Facial
D. Maxillary
241. Lymphatic drainage of pinna goes to:
A. Parotid node
B. Retroauricular node
C. Superficial cervical node along external jugular vein
D. All of the above .
242. Tegmen tympani is formed by:
A. Petrous part
B. Squamous part
C. Both of the above
D. Mastoid part
521. 243-glossopharyngeal nerve enters the middle ear through:
A. Roof
B. Floor
C. Anterior wall
D. Posterior wall
244. Processus cochleariformis contains:
A. Basal turn of cochlea
B. Tensor tympani tendon
C. Stapedius tendon
D. Apex of the cochlea
245. Central axis of cochlea is known as:
A. Spiral lamina
B. Modiolus
C. Processus cochleariformis
D. Crus commune .
246. Which of the following involve the floor of the mouth:
a- Ludwig's angina.
b- Infectious mononucleosis.
c- Peritonsillar abscess.
d- Glossitis.
247. Which is not correct for infectious mononucleosis:
a- it is viral.
b- Affect lymph glands.
c- Confirmed by Paul banner test.
d- There is increase in the count of monocytes.
248. all are present in the medial wall of tympanic cavity except:
a- oval window
b- Round window
c- Pyramid
d-promontory
e-tympanic part of facial nerve
e- There is increase in the count of lymphocytes
522. 249-Most benign tumor of frontal sinus
a- fibroma
b- polyp
c- angioma
d- osteoma
e- papilloma .
250-Which of the following is not the site for PARAGANGLIOMA?
a . Carotid bifurcation
b. Jugular foramen
c. Promontory in middle ear
d. Geniculate ganglion
251. Greisinger?s sign means:
A· Pain over the tempromandibular joint
B· Pain in the eye
C· Pain and tenderness over the mastoid
D· Pain and tenderness over the auricle
252. Fossa of Rosenmullar is a common site for:
A· Angiofibroma
B· Lipoma
C· Adenoid
D· Nasopharyngeal carcinoma
253. Early sign of nasopharyngeal carcinoma is:
A· Unilateral secretory otitis media
B· Trotter?s triad
C· Petrositis
D· Nasal obstruction
254. Reactionary bleeding after adenoidectomy is treated by:
A· Removal of the remnants and posterior pack
B· Antibiotics
C· Anterior nasal pack
D· Ligation of the bleeding vessel
523. 255-The usual cause of reactionary post-tonsillectomy bleeding is:
A· Slipped ligature
B· Wound infection
C· Tonsillar remnant
D· Early extubation
256. In multiple laryngeal papillomata, all of the following are true
accept:
A· Occurs in children
B· Affects only the glottic area
C· Recurrence is common
D· Best treated by laser surgery
257. Trotter?s triad is a clinical diagnosis of:
A· Nasopharyngeal angiofibroma
B· Nasopharyngeal cyst
C· Petrositis
D· Nasopharyngeal carcinoma
258. Posterior nasal packing is indicated in all of the following situations
except:
A· Severe posterior epistaxis
B· Antro-choanal polyp
C· Reactionary post-adenoidectomy bleeding
D· After removal of nasopharyngeal angiofibroma .
259. Griessinger?s sign is positive in:
A· Acute petrositis
B· Acute sinusitis
C· Acute labyrinthitis
D· Lateral sinus thromophelbitis
260. During total laryngectomy for malignancy, which of the following
maneuvers is not performed?
A. The strap muscles are divided and removed with the larynx.
B. The inferior constrictor is separated from the thyroid cartilage.
C. The vallecula is entered below the hyoid bone.
D. The trachea and esophagus are separated in an areolar plane,
which may beentered superiorly or inferiorly.
E. Tracheal transection occurs at least 2 cm below the tumor
524. 261-Total laryngogectomy for laryngeal cancer was first performed in
1873. by
A. Lange (United States).
B. Watson (Scotland).
C. Billroth (Austria).
D. Czerny (Austria).
E. Trotter (England).
262. Pharyngoesophageal reconstruction
A. is usually achievable with skin or dermal grafts.
B. seldom results in the resumption of swallowing.
C. is incompatible with speech rehabilitation.
D. is best accomplished by gastric transposition when the
esophagectomy extends low in the thorax.
E. cannot include neck skin reconstruction
263. Contraindications to total laryngopharyngectomy include
A. laryngeal spread of hypopharyngeal tumors.
B. extrapharyngeal spread.
C. superior mediastinal node metastasis.
D. deep invasion of the vertebral bodies.
E. vocal cord fixation
264. Which of the following has little effect on recorded auditory brain
stem response potentials during acoustic tumor removal?
A. filter types and settings
B. near- versus far-field recordings
C. depth of anesthesia
D. acoustic interference
E. rate of stimulation
265. Before beginning the removal of a cerebellopontine angle tumor,
the functional integrity of the facial nerve monitoring system should be
checked. Which method is preferred?
A. tapping the electrode tip
B. observing the stimulus artifact
C. stimulating the facial nerve in the mastoid segment
D. stimulating the muscle directly
E. measuring electrode impedances
525. 266-Which of the following cochlear components in most sensitive to
acoustic overstimulation?
A. inner hair cells
B. outer hair cells
C. spiral ganglion cells
D. dendritic endings of the cochlear nerve
267. The major effect of facial nerve monitoring during acoustic tumor
removal surgery is
A. decreased surgical time.
B. fewer postoperative complications.
C. increased proportion of cases with House-Brackmann grade I or II
results postoperatively
D. decreased incidence of House-Brackmann grade V and VI results
postoperatively.
E. reduced medicolegal risks.
268. Which of the following components of the auditory pathway is
suspected of contributing to the susceptibility of an individual to sound
overexposure effects?
A. cochlear efferent system
B. medial geniculate
C. auditory cortex
D. middle ear
269. The frequency region first affected by habitual exposure to
excessive sound is typically ___ kHz.
A. 0.5 to 2
B. 3 to 6
C. 6 to 8
D. 8 to 10
270. Herpes zoster oticus (Ramsay-Hunt syndrome)
A. is caused by primary varicella zoster virus infection.
B. produces bilateral vertigo and hearing loss mainly in the elderly.
C. produces unilateral facial weakness, ear pain, and occasional vertigo
or hearing loss mainly in the elderly.
D. produces unilateral loss of facial sensation, ear pain, vertigo, and
hearing loss mainly in the elderly.
E. none of the above
526. 271-Rubella causes the greatest fetal cochlear damage when the
maternal infection occurs
A. during the first trimester.
B. during the second trimester.
C. during the third trimester.
D. anytime during the pregnancy
272. The modern stapedectomy was first described in 1956 by
A. Toynbee.
B. Rosen.
C. Shea, Jr.
D. House
273. At times early stapes fixation of otosclerosis can be diagnosed
A. by seeing stapes fixation on physical examination.
B. by seeing an abnormality of the tympanic membrane.
C. when a maximum conductive hearing loss is present on audiogram.
D. when an on-off stapedial reflex can be seen on impedance
audiometry
274. What is the most common site of fixation of the stapes?
A. annular ligament
B. posterior crus
C. anterior crus
D. entire footplate
275. The facial recess provides access to the
A. internal auditory canal.
B. retrolabyrinthine air cells.
C. epitympanic space.
D. posterior mesotympanum.
E. mastoid antrum
276. Surface landmarks used for cortical mastoidectomy include the
A. sinodural angle of Citelli.
B. petrosquamous (Korner's) septum.
C. temporal line.
D. scutum
527. 277-Which of the following is an advantage of the modified radical
mastoidectomy over the intact wall procedure?
A. the ability to maintain the normal anatomy of the external ear canal
B. the ability to easily identify recurrent cholesteatoma
C. fewer episodes of postoperative otorrhea
D. superior hearing results
E. the fact that long-term follow-up is not necessary .
278. Which of the following is the most significant variable in successful
hearing restoration following ossiculoplasty?
A. size of the mastoid
B. presence or absence of middle-ear mucosa
C. eustachian tube function
D. type of prosthesis used
279. Which of the following is never required in the medial surface
tympanic membrane grafting technique?
A. deepithelialization of the surface of the tympanic membrane
remnant
B. lateralization of a medially rotated malleus handle
C. gelatin sponge packing in the eustachian tube
D. removal of a prominent bony protrusion of the anterior canal wall
280. A maximal conductive hearing loss occurs with which combination
of conditions?
A. ossicles intact, perforation centered over the round window
B. ossicles intact, total perforation
C. ossicular discontinuity, total perforation
D. ossicular discontinuity, intact tympanic membrane.
281. Which of the following temporal bone injuries carries the most
urgent need for surgical exploration?
A. transverse petrosa fracture with severe vertigo and nystagmus
B. longitudinal fracture with cerebrospinal fluid leakage and bleeding
from the external auditory canal
C. transverse temporal bone fracture
with immediate complete facial paralysis
D. penetrating tympanic membrane injury to the posterosuperior
quadrant with severe vertigo and nystagmus
E. triangular, trapdoor depressed tympanic membrane perforation
528. 282-The most useful radiographic study to evaluate the majority of
patients with trauma to the temporal bone is
A. axial computed tomography.
B. coronal computed tomography.
C. magnetic resonance imaging without contrast enhancement.
D. magnetic resonance imaging with contrast enhancement.
E. digital subtraction angiography .
283. Bezold's abscess results when mastoid infection
A. erodes the outer cortex of the mastoid tip.
B. in the zygomatic air cell erodes through the cortical bone at the
zygoma.
C. perforates the medial aspect of the mastoid tip.
D. perforates the outer cortex at the root of the zygoma into the
mandibular fossa
E. erodes the cortex through the area perforata
284. Which of the following is the poorest choice for first-line
antimicrobial therapy in acute otitis media?
A. erythromycin/sulfisoxazole
B. trimethoprim?sulfamethoxazole
C. clarithromycin or azithromycin
D. ciprofloxacin
E. amoxicillin
285. Infection spreads from the middle-ear cavity by which of the
following pathways?
A. periphlebitis or thrombophlebitis
B. bony erosion
C. direct extension through preexisting pathways
D. all of the above
E. a and c
286. In cholesteatomas, most bone resorption occurs because of
A. pressure necrosis.
B. the action of osteoblasts.
C. the action of osteoclasts.
D. the action of mononuclear phagocytes.
E. alkaline phosphatase
529. 287-Patients with aural cholesteatomas most commonly seek medical
advice because of
A. imbalance and vertigo.
B. chronic purulent drainage.
C. conductive hearing loss.
D. pain.
E. facial tics.
288-Children with a history of chronic otitis media with effusion have
mastoids that are
A. well pneumatized and sclerotic.
B. well pneumatized but not sclerotic.
C. poorly pneumatized and sclerotic.
D. poorly pneumatized but not sclerotic.
E. no different than those of normal children
289. Valsalva's and Politzer's tests of eustachian tube function assess
A. active tubal opening function but not tubal patency.
B. active tubal opening function and tubal patency.
C. tubal patency but not active tubal opening function.
D. tubal closing function but not tubal opening function.
E. tubal protective function but not tubal closing function
290. The infant eustachian tube differs developmentally from that in
adults in that the tube in infants is
A. longer.
B. shorter.
C. less compliant.
D. stiffer.
E. more efficient
291. The only muscle that actively opens the eustachian tube is the
A. levator veli palatini.
B. tensor veli palatini.
C. salpingopharyngeus.
D. lateral pterygoid.
E. medial pterygoid.
530. 292-Which of the following statements represents a characteristic of
the loudness of tinnitus?
A. Loudness matching with pure tones shows a close relationship to
subjective tinnitus loudness.
B. Loud tinnitus only occurs at pitch-matched frequencies with hearing
loss greater than 30 d
B. C. Loudness frequently fluctuates daily.
D. Parauditory tinnitus is perceived as louder than sensorineural
tinnitus.
E. Distressingly loud tinnitus is more characteristic of middle-aged
individuals than of the elderly
293. An ipsilateral retroauricular thrill in a patient with tinnitus suggests
A. a venous hum.
B. a carotid?cavernous fistula.
C. a glomus jugulare tumor.
D. a glomus vagale tumor.
E. an occipital arteriovenous malformation
294. Which of the following drugs is mostly likely not ototoxic?
A. bisphosphonate
B. vancomycin
C. carboplatin
D. deferoxamine
295. Which of the following drugs has the greatest vestibulotoxicity?
A. neomycin
B. netilmicin
C. streptomycin
D. kanamycin
296. Which of the following medications is known to cause cochlear
outer hair-cell damage?
A. furosemide
B. vancomycin
C. cisplatin
D. erythromycin
531. 297-In addition to facial nerve paralysis, the most common finding in
Ramsay-Hunt syndrome is
A. otalgia.
B. internuclear ophthalmoplegia.
C. downbeat nystagmus.
D. conductive hearing loss.
E. pulsatile tinnitus
298. Which of the following statements regarding otomycosis is true?
A. Systemic treatment is necessary.
B. Aspergillus and Candida are the most common responsible genera.
C. Invasive fungal infections are common in immunocompetent
patients.
D. The most common complaint is severe otalgia.
E. It occurs most commonly in diabetic
299. Which of the following statements regarding necrotizing external
otitis are true?
A. Staphylococcus is the predominant pathogen.
B. It occurs only in elderly, diabetic patients.
C. Diagnosis is made by gallium-67, technetium-99m, and high-
resolution computed tomography.
D. Surgical intervention is usually limited to minimal debridement.
E. Mortality is 75%.
300. The optimum surgical approach for the auditory brain stem
implant is the
A. middle cranial fossa.
B. retrosigmoid.
C. suboccipital.
D. transcochlear.
E. translabyrinthine
301. The target region for the auditory brain stem implant is the
A. interstitial nucleus of Cajal.
B. dorsal and ventral cochlear nuclei.
C. zona inserta of the cochlear nerve.
D. roof of the fourth ventricle.
E. superior and medial vestibular nuclei.
532. 302-A 30-year-old woman in good health with which of the following
would be the best candidate for an Audiant bone-anchored hearing aid?
A. bilateral sloping moderate to profound sensorineural hearing loss
B. bilateral flat moderate sensorineural hearing loss
C. bilateral 60-dB conductive hearing loss from EAC atresia
D. bilateral moderate CHL from ossicular discontinuity
E. bilateral severe mixed hearing loss from otosclerosis
303. In children 6 to 24 months of age, the technique most commonly
used to assess hearing is
A. behavioral observation audiometry.
B. visual reinforcement audiometry.
C. conditioned play audiometry.
D. conventional audiometry.
E. auditory brain stem response audiometry
304. Facial nerve stimulation during cochlear implant use is most likely
to occur in patients with deafness from
A. large vestibular aqueduct syndrome.
B. aminoglycoside ototoxicity.
C. temporal bone fracture.
D. congenital cytomegalovirus infection.
E. autoimmune inner-ear disease
305. The most accurate information needed for planning reconstruction
of the ossicular chain is provided by
A. tympanometry.
B. computed tomography scanning.
C. pure tone and speech audiometric testing.
D. impedence testing.
E. intraoperative ossicular assessment
306. Which of the following should raise the greatest suspicion that a
CHL on the audiogram is not, in fact, conductive at all?
A. an absent ipsilateral acoustic reflex
B. a word understanding score of 64%
C. a type A tympanogram
D. a type Ad tympanogram
E. a type C tympanogram
533. 307-The transmission of sound energy through the middle ear in
humans is influenced most by the
A. areal ratio of the round-to-oval window.
B. ossicular lever ratio.
C. pars flaccida.
D. peripheral third of the pars tensa.
E. central two thirds of the pars tensa.
308. The use of intratympanic aminoglycoside antibiotics for the control
of vertigo in Ménière's disease
A. is beneficial only with complete ablation of ipsilateral peripheral
vestibular function.
B. has a lower incidence of postoperative hearing loss.
C. decreases endolymph volume secondary to toxic effects on the
dark cells of the vestibular labyrinth.
D. is restricted to bilateral disease.
E. is contraindicated in the elderly.
309. Which of the following is thought to be of greatest importance in
the etiology of Ménière's disease?
A. failure of the resorption limb of radial endolymph flow.
B. failure of the resorption of longitudinal endolymph flow.
C. immune complex reactions in the stria vascularis.
D. enlargement of the vestibular aqueduct and endolymphatic sac.
E. decreased endolymph production by the dark cells of the vestibular
labyrinth .
310. Shrapnell's membrane is attached directly to the squama of the
temporal bone at the
A. suprameatal spine.
B. petrosquamous suture.
C. tympanic sulcus.
D. notch of Rivinus.
E. cribriform area
311. The most common cause of inherited congenital deafness is
A. the oculoauriculovertebral spectrum.
B. autosomal recessive inheritance.
C. Stickler syndrome.
D. Usher's syndrome.
E. branchio-otorenal syndrome
534. 312-Which test reflects vestibulospinal function?
A. headshake nystagmus
B. electronystagmography
C. rotatory chair testing
D. tandem walking
E. positional testing
313. Thyroid cancer
A. does not occur in multinodular goiter.
B. is frequently undetectable by palpation.
C. constitutes 50% of endocrine malignancies.
D. is found as an occult malignancy in 1% of autopsies.
E. is more frequent than benign thyroid lesions in irradiated thyroid
glands .
314. What is the most common ectopic location for a missing superior
parathyroid gland?
A. intrathymic
B. carotid sheath
C. intrathyroidal
D. superior mediastinal
E. retroesophageal?retropharyngeal area
315. Which pharyngeal pouch gives rise to the superior parathyroid
glands?
A. first
B. second
C. third
D. fourth
E. fifth
316. Inspiratory stridor tends to occur when there is an obstruction
A. above the level of the vocal cords.
B. at the level of the vocal cords.
C. below the level of the vocal cords.
D. in the nasal airway.
E. all of the above
535. 317-Otolaryngology patients with reflux disease appear to differ from
most gastroenterology patients in that they
A. are obese.
B. have dysphagia.
C. have heartburn.
D. have regurgitation.
E. have daytime reflux
318. Which test is the most sensitive for the diagnosis of
extraesophageal reflux?
A. barium esophagography
B. esophagoscopy and biopsy
C. radionuclide scanning
D. ambulatory pH monitoring
E. Bernstein acid perfusion test
319. Which technique can be used for cricopharyngeal spasm after total
laryngectomy to improve voice?
A. pharyngeal plexus neurectomy
B. cricopharyngeal myotomy
C. botulinum toxin injection of the pharynx
D. all of the above .
320. The only absolute contraindication to performing a secondary
tracheoesophageal puncture for speech rehabilitation is
A. stomal stenosis.
B. pharyngeal stenosis.
C. decreased vision.
D. manual dexterity.
E. severe chronic obstructive airway disease
321. In rheumatoid arthritis involving the larynx,
A. onset is slow and progressive, without airway emergencies.
B. cricoarytenoid joint arthritis occurs in the majority.
C. laryngeal muscles are unaffected.
D. rheumatoid nodules do not occur.
E. symptoms are often a presenting feature of the disease
536. 322-The best indication for use of the Co2 laser in laryngeal surgery is
A. excision of benign vocal fold polyps.
B. the treatment of recurrent respiratory papillomas.
C. vaporization of a vocal fold lesion.
D. excision of an anterior commissure vocal fold carcinoma .
323. Which pharyngoesophageal reconstruction technique results in the
highest rateof postoperative pharyngocutaneous fistula?
A. free jejunal transposition
B. pectoralis major myocutaneous flap
C. gastric pull-up
D. radial forearm free flap
E. deltopectoral flap
324. Which technique is most useful for reconstruction after total
laryngopharyng
-oesophagectomy?
A. free jejunal transposition
B. radial forearm free flap
C. pectoralis major myocutaneous flap
D. lateral thigh fasciocutaneous flap
E. gastric pull-up
325. In treating laryngeal granulomas,
A. it is essential to remove the precipitating factor.
B. speech therapy is usually not required.
C. patients should not be treated for reflux if radiographic studies are
negative.
D. excision with the carbon dioxide laser should be avoided
326. Which of the following statements regarding surgical treatment of
vocal fold cysts is true?
A. Vaporization of the cyst and overlying mucosa with the carbon
dioxide laser is the best treatment.
B. Marsupialization using microlaryngeal scissors is the best treatment.
C. Endoscopic microaspiration of cysts is the easiest and most
effective treatment.
D. Excision preserving the overlying mucosa is the ideal treatment.
E. Rate of recurrence is low regardless of the surgical treatment
537. 327-Preepiglottic space involvement by supraglottic carcinoma is
associated with
A. cricoid cartilage involvement.
B. a positive Delphian node.
C. thyroid cartilage involvement.
D. level IV node involvement.
E. bilateral cervical metastases.
328. In treating early vocal fold cancer,
A. carbon dioxide laser excision offers a higher cure rate than does
radiotherapy.
B. carbon dioxide laser excision is more effective than is radiation
therapy if the tumor involves the anterior commissure.
C. radiation therapy is associated with less morbidity than is endoscopic
excision.
D. laser excision should not be attempted in cases of recurrence.
E. endoscopic excision offers a one-session treatment as opposed to
multiple sessions with radiotherapy
329. The best initial treatment of infantile subglottic hemangiomas is
A. carbon dioxide laser vaporization.
B. tracheotomy and open excision
C. conservative treatment with steroids and observation.
D. the Nd:YAG laser.
330. The most sensitive test to assess arytenoid mobility in patients
with unilateal or bilateral vocal fold motion impairment is
A. laryngeal electromyography.
B. flexible fiberoptic laryngoscopy.
C. flow?volume loop and spirometry.
D. dynamic laryngeal computed tomography.
E. direct laryngoscopy and arytenoid palpation .
331. Which of the following is the least acceptable treatment for vocal
nodules?
A. precise vaporization with the microspot Co2 laser set a 0.5 W
B. vocal fold stripping
C. precise removal with forceps and microscissors
D. voice therapy from a speech pathologist
538. 332-Which kind of laryngeal cyst has been reported in association with
larynx cancer?
A. anterior saccular
B. lateral saccular
C. epidermoid inclusion
D. mucus retention
333. The best method to assess both the etiology and degree of
aspiration is
A. videolaryngoscopy.
B. pharyngeal manography.
C. scintigraphy.
D. dynamic magnetic resonance imaging.
E. videofluoroscopic swallowing study
334. Vocal nodules
A. often require surgical therapy.
B. always result in dysphonia.
C. are congenital.
D. are synonymous with vocal cord cysts.
E. usually respond to medical and behavioral therapy
335. Optimal results after vocal fold surgery may be best achieved by
including
A. postoperative absolute voice rest.
B. smoking cessation.
C. antireflux therapy.
D. preoperative and postoperative voice therapy.
E. perioperative steroids
336. Which of the following is a contraindication for MRI ?
A. cardiac pacemaker
B. metallic cochlear implant
C. cerebral aneurysm clip
D. all of above
337. Retropharyngeal space enlargement can occur in
A. retropharyngeal abscess.
B. lymphoma.
C. cystic hygroma.
D. all of the above
539. 338-Laryngeal stroboscopy is
A. unable to assist in the diagnosis of glottic cancer.
B. unable to reveal vocal scanning effects.
C. a series of light flashes coordinated with fundamental vocal
frequency.
D. very useful in patients with vocal fold paralysis.
E. only able to demonstrate the vertical mucosal wave.
339. A chyle leak can be treated by all of the following except
A. bed rest.
B. continued suction drainage.
C. a long-chain fatty-acid diet.
D. pressure dressings.
E. A chyle leak can be treated by all of the above
340. Which structure is situated between the aryepiglottic fold and the
vestibular fold?
A. conus elasticus
B. saccule
C. triangular membrane
D. ventricle
E. quadrangular membrane
341. Patients with cricothyroid joint arthritis frequently complain of
A. dysphagia.
B. aspiration.
C. hoarseness.
D. odynophagia.
E. neck pain
342. Lymphomas of the head and neck present in all of the following
areas except
A. the cervical nodes.
B. Waldeyer's ring.
C. the thyroid gland.
D. the subdermal layer of skin.
E. the orbit.
540. 343-Which of the following lymph node groups is removed in
posterolateral neck dissection?
A. suboccipital
B. perilateral
C. postauricular
D. periparotid
E. a and c
344. Which nerves are routinely preserved in supraomohyoid neck
dissections?
A. motor branches of the cervical plexus
B. greater auricular nerve
C. ansa cervicalis
D. sensory branches of the cervical plexus
E. a, b, and d
345. Neurofibromas differ from schwannomas in that
A. schwannomas are not encapsulated.
B. neurofibromas can be easily resected from the nerve of origin.
C. neurofibromas do not undergo sacomatous transformation.
D. neurofibromas are not encapsulated
346. von Recklinhausen's disease is characterized by
A. autosomal dominant inheritance.
B. more than five café-au-lait spots on the body.
C. positive family history in around 50% of cases.
D. possible association with spinal bifida.
E. all of the above
347. Which nodes are most frequently involved in squamous cell
carcinoma of the upper aerodigestive tract?
A. external jugular
B. upper jugular
C. submandibular
D. supraclavicular
E. preauricular
541. 348-The best incision to explore the carotid sheath for a unilateral
penetrating neck injury is a(n)
A. modified Conley incision.
B. lateral cervical incision along the anterior sternocleidomastoid
muscle.
C. MacFee incision.
D. H incision.
E. Schobinger incision
349. Which zone of the neck has the most difficult surgical access?
A. base of the skull region
B. midcervical region
C. lower cervical region below the cricoid
D. posterior neck triangle
E. anterior cervical triangle
350. The anatomic boundaries of zone III of the neck are from the
A. hyoid to the base of the skull.
B. superior border of the thyroid cartilage to the base of the skull.
C. hyoid to the mandible angle.
D. mandible angle to the base of the skull.
E. clavicle to the cricoid
351. The prevertebral fascia envelops all of the following except the
A. vagal nerve.
B. brachial plexus.
C. sympathetic trunk.
D. cervical nerve plexus.
E. phrenic nerve
352. Which of the following surgical principles for dealing with deep
neck infection is false?
A. Establishing an airway is the first priority.
B. Wide debridement of necrotic fascia and skin is necessary in
treating patients with cervical fasciitis.
C. External drainage of all deep neck abscesses is necessary.
D. Normal anatomy is often distorted.
E. Ligation of thrombosed vessels may be necessary
if thrombosis has occurred
542. 353-Trismus associated with deep neck infection is most indicative of
A. peritonsillar abscess.
B. parapharyngeal abscess.
C. parotitis.
D. submandibular abscess.
E. infection tracking along the carotid sheath
354. Nasopharyngeal angiofibroma ?
A. is a common tumor of the head and neck.
B. arises from the posterolateral wall of the roof of the nose.
C. is seen only in patients in their teenage years or younger.
D. requires angiography for diagnosis.
E. is unlikely to be resectable through a lateral rhinotomy approach
355. Apnea is defined as a cessation of air flow at the nostrils and
mouth for _ seconds.
A. 5
B. 10
C. 15
D. 20
356. What is the predominant blood supply to the platysmal
myocutaneous flap?
A. submental branch of the facial artery
B. transverse cervical artery
C. platysmal branch of the occipital artery
D. ascending cervical artery
357. Nodal metastases in nasopharyngeal carcinoma
A. are rarely the presenting symptom.
B. indicate a worse prognosis if bilateral.
C. indicate a worse prognosis if low in the neck.
D. are not treated surgically.
E. should be confirmed by open biopsy prior to definitive therapy.
543. 358-Nasopharyngeal carcinoma
A. commonly presents as serous otitis media.
B. behaves more aggressively in Chinese populations.
C. has strong human leukocyte antigen associations in non-Chinese
populations.
D. may be excluded on nasopharyngeal examination if the mucosa is
macroscopically normal.
E. invades the skull base in only 10% of cases.
359-Residual disease after resection of nasopharyngeal angiofibroma
A. may remain asymptomatic.
B. has a natural history of relentless progression.
C. will eventually require a secondary form of treatment
D. is often fatal.
E. has a worse outcome in older patients
360. Otalagia may occur via cranial nerve
A. IX, from the glottic larynx.
B. IX, from the anterior lateral tongue.
C. X, from the hard palate.
D. IX, from the retromolar trigone.
E. V, from the oropharynx
361. What is the appropriate treatment for a T1N0 midline soft palate
lesion?
A. complete local excision and no postoperative radiation therapy if
margins are clear
B. complete local excision and postoperative radiation therapy to the
primary lesion if the margins are positive
C. radiation therapy to the primary lesion and both necks
D. radiation therapy to the primary lesion only
E. radiation therapy to the primary lesion and one side of the neck
362. The factor that is least likely to influence prognosis in squamous
cell carcinoma of the oropharynx is
A. tumor size.
B. depth of invasion.
C. histologic differentiation.
D. blood transfusion.
E. decreased proportions of cytotoxic suppressor cells
544. 363-Which type of malignant tumor may have a bilateral presentation?
A. mucoepidermoid carcinoma
B. adenoid cystic carcinoma
C. expleomorphic carcinoma
D. adenocarcinoma
E. acinic cell carcinoma
364. What is the most common presenting symptom of a malignant
parotid tumor?
A. facial nerve paralysis
B. skin ulceration
C. nodal metastasis
D. painless swelling
E. pain
365. Which of the following does not influence the timing and
performance of facial reanimation?
A. donor site morbidity
B. proximal and distal nerve integrity
C. age
D. inability to close the eyelid
E. patient expectations
366. What is the most successful approach for repair of most cribriform
cerebrospinal fluid leaks?
A. intranasal
B. external nasal
C. transantral
D. intradural
E. extradural
367. Cerebrospinal fluid leak may be confidently diagnosed by all of the
following means except
A. B2 transferrin.
B. computed tomography cisternography.
C. intrathecal dye injection.
D. intrathecal radio-tracer injection.
E. glucose oxidase test paper
545. 368-Which of the following would not explain the development of a
delayed
cerebrospinal fluid leak?
A. delayed increase in intracranial pressure
B. lysis of a clot in areas of dehiscence
C. resolution of edema
D. neural degeneration of olfactory fibers
E. wound maturation
369. The technique of removal of the intrasinus septum of the frontal
sinus is termed
A. the Ridell procedure.
B. the Lothrop (Chaput-Mayer) procedure.
C. Killian's procedure.
D. Lynch frontoethmoidectomy.
E. Kummel-Beck trephination
370. One contraindication to the Caldell-Luc operation is
A. foreign body in the antrum.
B. mucocele.
C. unerupted teeth in children.
D. antral cysts
371. During inspiration the main current of airflow in a normal nasal
cavity is through:
A. Middle part of the cavity in middle meatus in a parabolic curve
B. Lower part of the cavity in the inferior meatus in a parabolic curve
C. Superior part of the cavity in the superior meatus
D. Through olfactory area
372. Which o fthe following is the most common etiological agent in
paranasal sinus mycoses?
a) Aspergillus spp
b) Histoplasma
c) Conidiobolus coronatus
d) Candida albicans
546. 373-audiogram in early meniere's disease shows:
a-notch at 2 kHz in bone conduction.
b-notch at 4 kHz in air conduction.
c-a flat curve.
d-a rising curve
374. Melkersson-Rosenthal syndrome includes all except:
a-facial paralysis.
b-fissured tongue.
c-circumoral oedema.
d-sarcoidosis
375. In ramsay hunt syndrome,vesicular eruptions are seen in all the
following areas except?
a-concha.
b-posteromedial surface of pinna.
c-soft palate.
d-tragus and surrounding skin
376. recurrent facial paralysis is seen in all except:
a-acoustic neuroma.
b-diabetes.
c-sarcoidosis.
d-cholesteatoma.
377. landmark used for identification of geniculate ganglion of facial n.
is:
a-oval window.
b-processus cochleariformis.
c-pyramid.
d-digastric ridge
547. 378- Commonest complication of CSOM is
a) conductive deafness
b) Meningitis
c) temporal lobe abscess
d) Cholesteotoma
379. Unilateral nasal obstruction in a child is most
often due to
a) foreign body
b) chaonal atresia
c) polyps
d) fungal infection
380. Regarding multiple papillomas in the larynx which of the following
statements are correct:
a) common in infants and children
b) viral in origin
c) vocal cords usual site
d) treatment is removal by direct laryngoscopy
e) All are correct
548. 381- Hard elongated swelling in the tonsillar fossa
after tonsillectomy is:
a) thrombosed vein
b) tonsilolitth
c) elongated styloid process
d) calcified esophagus
382. Treatment of choice for otosclerosis is
a) Stepedectomy
b) fenestration
c) Hearing aid
d) taympanoplasty
383. Multi perforations of the tympanic membrane is
characteristic of otitis media due to
a) Staphylococcal
b) Strepta
c) tuberculous
d) Pneumococcal
549. 384- Carcinoma larynx with no neck secondaries is
treated by
a) laryngectomy
b) Laryngectomy and radical neck dissection
c) Radiation
d) Radiation & laryngectomy
385. Treatment of cholesteatoma of the middle ear
in a 6 year old
a) Radical mastoidectomy
b) modified radical mastoidectomy
c) simple mastoidectomy
d) None
386. The earliest symptom of acoustic neuroma is
a) deafness
b) Tinnitus
c) Vertigo
d) facial weakness
550. 387- Tonsils develop embryologically from
a) First pharyngeal pouch
b) Second pharyngeal pouch
c) Third pharyngeal pouch
d) Fourth phafyrfgea4pouch
388. Lancinating pain around the tonsils during eating
is indicative of:
a) Trigeminal neuralgia
b) Glossophyngeal neuralgia
c) facial neuralgia
d) None of the above
389- Polyp which has a mulberry appearance and
bleeding on touch is due to
a) Malignancy
b) Rhinosporiodisis
c) Antrochoanal polyp
d) Nasopharyngeal angiofibroma
551. 390-Which type of diverticulum is common in the
oesophagus
a) traction type
b) pulsion type
c) rolling hernia
d) none
391. Which collagen disease most commonly affects
the esophagus
a) Scleroderma
b) SLE
c) Polyarteritis nodosa
d) Wegners
392. Unsafe otitis media in a child with high fever
and convulsion, the diagnosis is
a) temporal lobe abscess
e) cerebellar abscess
b) meningitis
c) lateral sinus thrombosis
552. 393-Treatment of maxillary antral carcinoma (T3 No)
is
a) Only surgery
b) surgery and radiotherapy
c) chemotherapy of radiotherapy
d) radiotherapy only
394. Labyrinthine artery is a branch of
a) Internal carotid artery
b) Basilar artery
c) Posterior cerebellar artery
d) Anterior inferior cerebellar artery
395. Apple-jelly nodules on the nasal septum are
found in cases of:
a) Tuberculosis
b) Lupus vulgaris
c) Syphilis
d) Scleroma
e) None of the above
553. 396-Costen's syndrome refers to neuralgia
originating in the:
a) Temporomandibular joint
b) Optic foramen
c) Meatal foramen
d) Sphenopalatine ganglion
397. Areas of carcinoma of oral mucosa can be
identified by staining with:
a) 1 % zinc chloride
b) 2 % silver nitrate
c) Gentian violet blue
d) 2 % toluidine blue
398. Gustatory sweating and flushing (Frey's
syndrome) follows damage to the:
a) Trigeminal nerve
b) Facial nerve
c) Glossopharyngeal nerve
d) Vagus nerve
e) Auriculotemporal nerve
554. 399-Hyposthesia of the posterior aspect of the
external auditory canal may be an early sign of:
a) Trigeminal neuralgia
b) Costen?s syndrome
c) Lateral sinus thrombosis
d) Multiple sclerosis
e) Acoustic neuroma
400. Hyposthesia of the posterior aspect of the
external auditory canal may be an early sign of:
a) Trigeminal neuralgia
b) Costen?s syndrome
c) Lateral sinus thrombosis
d) Multiple sclerosis
e) Acoustic neuroma
401. Medication which may prevent rapid progress of
cochlear otosclerosis is
a) steroids
b) antibiotics
c) fluorides
d) vitamins
555. 402-Conditions that may cause esophageal strictures include all of the
following except
A. Behçet?s syndrome.
B. gastroesophageal reflux.
C. caustic ingestion.
D. polymyositis.
E. Crohn?s disease.
403. A white boy presents with a history of chronic sinusitis, pulmonary
infection, and malabsorption. Which of the following tests is the most
specific for his disease?
A. sweat chloride
B. c-ANCA
C. erythrocyte sedimentation rate
D. angiotensin converting enzyme level
E. rheumatoid factor
404. Which of the following therapeutic agents can be most
successfully used in the acute management of hereditary angioedema?
A. epinephrine
B. corticosteroid
C. antihistamine
D. C1-INH concentrate
E. attenuated androgen
405. The treatment of choice for verrucous carcinoma is
A. surgical resection followed by chemotherapy.
B. surgical resection followed by radiation therapy.
C. chemotherapy followed by radiation therapy.
D. radiation therapy followed by chemotherapy.
E. surgical resection.
406. A patient presents with bilateral cauliflower ear deformity, saddle
nose deformity, and stridor. Which of the following therapeutic agents
may be useful?
A. cidofovir
B. interferon
C. dapsone
D. attenuated androgens
E. C1-INH concentrate
556. 407-The ?gold standard? for treatment of advanced Wegener?s
granulomatosis is glucocorticoids and
A. trimethoprim and sulfamethoxazole.
B. methotrexate.
C. quinolone antibiotics.
D. cyclophosphamide.
E. dapsone.
408. The virus most commonly causing mumps (epidemic parotitis) is a
A. coxsackievirus.
B. poxvirus.
C. paramyxovirus.
D. herpesvirus.
E. papovavirus.
409. The fascia covering the parotid and submandibular glands
originates from which layer?
A. the superficial cervical fascia
B. the visceral or middle layer of the cervical fascia
C. the superficial layer of the deep cervical fascia
D. the superficial musculoaponeurotic system
E. the carotid sheath
410. Regional metastases to cervical lymph nodes decrease 5-year
survival rates by
A. 10%.
B. 20%.
C. 50%.
D. 80%.
E. 90%.
411. The most common synchronous secondary primary tumor in
patients with laryngeal carcinoma is
A. floor of the mouth carcinoma.
B. bronchogenic carcinoma.
C. base of the tongue carcinoma.
D. lymphoma.
E. esophageal carcinoma
557. 412-Which procedure is not indicated for vocal fold paralysis with
aspiration?
A. tracheostomy
B. laryngectomy
C. arytenoidectomy
D. medialization thyroplasty
E. Teflon injection
413. Isolated laryngeal candidiasis is usually secondary to
A. prolonged antimicrobial use.
B. systemic corticosteroid use.
C. inhaled corticosteroid use.
D. insulin-dependent diabetes.
414. Which section of the tracheobronchial system exhibit seromucous
glands?
A. Segmental bronchi
B. bronchioles
C. bronchioles terminales
D. bronchioles respiratory
E. alveolar ducts
415. Alveolar type II cells contain many lamellar bodies throughout
their cytoplasm containing:
A. Lysosomes
B. Phagocytosed debris
C. Secretory material
D. Mitochondria
E. infection with human immunodeficiency virus
416. The function of the ciliated epithelial cells of the respiratory
mucosa is:
A. secrete mucus
B. move mucus toward the pharynx
C. move macrophages toward the alveoli
558. 417-Concerning the thymus:
A. Lymphoid nodules form thymic cortex.
B. Epithelial reticular cells form Hassal?s corpuscles.
C. T cells migrate into medulla to become immunologically competent
D. Macrophage are essential component of blood thymus barrier.
418. Which of the following is thought to function in preventing
immune response against self antigens:
A. B cells.
B. T memory cells.
C. T helper cells.
D. T suppressor cells.
419. Which of the following are important factors in considering a
temporalis muscle transposition procedure?
a. Airway structure
b. Appearance of nasolabial structures
c. Type of smile on unaffected side
d. b and c
e. All of the above
D. are sensory cells
420. The best treatment of postoperative trismus is
a. Temporomandibular joint (TMJ) resection
b. TMJ prosthesis
c. Stretching therapy
d. Laser scar ablation
e. Steroid injection into the pterygoid muscles
421. Sagging of the postero-superior wall of the EAC occurs in:
a. Acute mastoiditis.
b. Furunculosis of the external ear.
c. Otitis media.
d. Petrositis.
422. Gradinigo syndrome occurs in:
a. Acute mastoid abscess.
b. Acute petrositis.
c. Chronic otitis media.
d. Secretory otitis media.
559. 423-Tobey-Ayer's test is a characteristic sign in:
a. Brain abscess.
b. Lateral sinus thrombosis.
c. Extradural abscess.
d. Meningitis.
e. Cavernous sinus thrombosis.
424. The fluids presents in secretory otitis media is:
a. Mucopurulent.
b. Serosanguinous.
c. Exudates.
d. Transudates.
e. Mixture of exudates & transudates.
425. Longitudinal fracture of the temporal bone may be associated with
all of the following EXCEPT:
a. LMNL facial palsy.
b. Traumatic perforation of the tympanic membrane.
c. Conductive hearing loss.
d. Profound hearing loss.
426. ABR "Auditory Brain stem Response" is used in:
a. Test of hearing in malingering.
b. Test of hearing in retrochoclear lesion.
c. Detection of acoustic neuroma.
d. All of the above.
427. The most accurate diagnostic test to detect degeneration of the
facial nerve:
a. Nerve excitability test.
b. Electromyography.
c. Electroneurography.
d. Stapedial reflex.
560. 428-The most common vertigo is:
a. Acoustic neuroma.
b. Ototoxicity.
c. Meniere's disease.
d. Benign paroxysmal positional vertigo.
429. The commonest organism in malignant otitis externa:
a. Streptococci.
b. Pneumococci.
c. Pseudomonas aeroginosa.
d. Moraxella catarrhalis.
430. Mixed hearing loss may be caused by one of the following:
a. Otosclerosis.
b. Meniere's disease.
c. Ear wax.
d. Acoustic neuroma.
431. Fluctuant SNHL usually occurs in:
a. Presbyacusis.
b. Meniere's disease.
c. Otosclerosis.
d. All of the above.
432. Nystagmus & vertigo induced by pressure on the tragus is a sign
of:
a. Fistula complicating cholesteatoma.
b. Benign paroxysmal vertigo.
c. Vestibular neuritis.
d. Cholesteatoma only.
433. The earliest symptom in a case with cholesteatoma that indicates
intracranial complication is:
a. Persistent headache.
b. Facial palsy.
c. SNHL.
d. Squint.
561. 434-A false +ve fistula test is due to:
a. Labyrinthine fistula with dead ear.
b. Cholesteatoma bridging an inner ear fistula.
c. Hyper mobile footplate of the stapes.
d. All of the above.
435. The concept that the facial nerve supplies the auricle is related to:
a. Ramsy-Hunt syndrome.
b. Jugular foramen syndrome.
c. Horner's syndrome.
d. Bell's palsy.
436. Topognostic test is used in the assessment of facial paralysis
include all the following EXCEPT:
a. Schirmer test.
b. Stapedial reflex.
c. Electroneurography.
d. Gustatory test.
437. All are true about rhinoscleroma EXCEPT:
a. It is endemic in Egypt.
b. It is a disease of low immunity & low resistance.
c. It runs in families due to genetic inheritance.
d. It is sub-epithelial inflammatory granuloma.
438. Mikulicz cell is a characteristic histological finding in:
a. Rhinoscleroma.
b. Rhinosporodosis.
c. Aspergillosis.
d. Sarcoidosis.
439. Perforation of bony part of the nasal septum occurs in:
a. Sarcoidosis.
b. Rhinoscleroma.
c. Tuberculosis.
d. Syphilis.
562. 440-Spontaneous recovery is usual in:
a. Rhinosporodosis.
b. Sarcoidosis.
c. Tuberculosis.
d. Syphilis.
441. The causative agent of rhinoscleroma is:
a. Sporozoon.
b. Low virulent T.B bacillus.
c. Treponema Ballidum.
d. Gram ?ve short capsulated diplobacillus.
442. The causative agent of lupus vulgaris is:
a. Sporozoon.
b. Low virulent T.B bacillus.
c. Treponema Ballidum.
d. Gram ?ve short capsulated diplobacillus.
443. All the following lines of treatment could be applied in
rhinoscleroma EXCEPT:
a. Rifampicin.
b. Cytotoxic drugs.
c. Surgery to canalize the stenosed canal.
d. Laser surgery.
444. Unilateral polypoidal mass arising from the lateral wall of the nose
in 55 years old man is most probably:
a. Inverted papilloma.
b. Rhinoscleroma.
c. Allergic nasal polyp.
d. Antrochoanal polyp.
445. Rhinoscleroma characterized by the following EXCEPT:
a. Hard-like nodules.
b. Histopathology shows Mikulicz cells.
c. Responses to rifampicin.
d. Ulceration of the surrounding tissue.
563. 446-Resistant epistaxis from below the middle turbinate requires
ligation of:
a. The anterior ethmoidal artery.
b. The sphenopalatine artery.
c. The maxillary artery.
d. The internal jugular vein.
447. Nasopharyngeal carcinoma cause Horner's syndrome as a result of
infiltration of:
a. 3rd cranial nerve.
b. 5th cranial nerve.
c. 7th cranial nerve.
d. Cervical sympathetic chain.
448. Surgery is contraindicated for ca.of maxillary sinus if X-ray refeal
of invovement of:
A. Floor of the orbit.
B. Foramen ovale.
C. Lateral antral wall.
D. Lamina papyracea.
E. Lateral ptyregoid plate
449. The best results in the treatment of advance ca.maxilla are
achieved by:
A. Surgery alone.
B. Radiation alone.
C. Surgery and radiation.
D. Surgery and chemotherpy.
450. In sudden recent unilateral hearing loss with vertigo:
A. Betahestine is the treatment of choice.
B. should be treated as an emergency.
C. Injectable steroid should be given for all patients.
D. The patient should be refered for out patient for investigations.
E. Antimicrobial treatment is of no value.
564. 451-Primary atrophic rhinitis is characterized by all of the following
except:
A. Recurrent epistaxis
B. Anosmia
C. Bad odor smelled by the neighbors
D. Affects male only
E. Laser resection
452. Functional endoscopic sinus surgery is the operation of
choice in all of the following except:
A. Chronic sinusitis
B. Mucocele of the paranasal sinuses
C. Nasal polyposis
D. Twisted nose
453. An adult presented with acute onset dysphagia, edema
of the uvula and palate, medially pushed tonsil and trismus is
most likely to have:
A. Parapharyngeal abscess
B. Acute retropharyngeal
C. Quinsy
D. Chronic retropharyngeal abscess
454. The following are signs of chronic tonsillitis except:
A. Enlarged cervical lymph nodes
B. Inequality of the size of the tonsils
C. Pus in the tonsillar crypts
D. Edema of the uvula
455. In multiple laryngeal papillomata, all of the following are
true accept:
A. Occurs in children
B. Affects only the glottic area
C. Recurrence is common
D. Best treated by laser surgery
565. 456-T.B of the larynx mostly affects:
A. Posterior part of the larynx
B. Supraglottis
C. Anterior part of the larynx
D. Subglottis
457. Safe type of chronic suppurative otitis media is characterized by:
A. Scantly offensive ear discharge
B. Profuse mucopurulent ear discharge
C. Marginal perforation
D. All of the above
458. Absent laryngeal click may suspect:
A. Supraglottic carcinoma
B. Pyriform fossa tumor
C. Subglottic cancer
D. Post-cricoid carcinoma
459. Myringotomy is indicated in:
A. Secretory otitis media
B. Acute otitis media with bulging drum
C. Otitic barotrauma
D. All of the above
460. The most serious complication following adenotonsillectomy is:
A. Reactionary hemorrhage
B. Respiratory obstruction
C. Respiratory infection
D. Non of the above
461. All the following are signs of chronic tonsillitis except:
A. Inequality of size
B. Congested anterior pillars
C. Presence of crypts over the medial tonsillar surface
D. Enlarged cervical lymph nodes
566. 462-Posterior nasal packing is indicated in all of the following situations
except:
A. Severe posterior epistaxis
B. Antro-choanal polyp
C. Reactionary post-adenoidectomy bleeding
D. After removal of nasopharyngeal angiofibroma
463. Shwartz?s sign may be positive in:
A. Chronic suppurative otitis media
B. Atelactatic middle ear
C. Meniere?s disease
D. Otosclerosis
464. Griessinger?s sign is positive in:
A. Acute petrositis
B. Acute sinusitis
C. Acute labyrinthitis
D. Lateral sinus thromophelbitis
465. Recurrent laryngeal nerve is motor to all of the intrinsic muscles of
the larynx except:
A. Vocalis muscle
B. Posterior cricoarytenoid muscle
C. Lateral cricoarytenoid muscle
D. Cricothyroid muscle
466. The only abductor muscle in the larynx is:
A. Sternothyroid muscle
B. Lateral cricoarynoid muscle
C. Cricothyroid muscle
D. Posterior cricoarynoid muscle
467. Cortical mastoidectomy is indicated in:
A. acute mastoiditis not responding to medical treatment
B. Bezold?s abscess
C. mastoid abscess
D. all of the above
567. 468-Facial nerve gives secretomotor supply to:
A. parotid gland
B. submandibular salivary gland
C. pituitary gland
D. thyroid gland
469. All the following are possible complications of acute sinusitis
except:
A. temporal lobe brain abscess
B. cavernous sinus thrombosis
C. orbital cellulitis
D. osteomylaitis
470. One of the signs of otogenic cerebellar abscess is:
A. hemiplegia
B. hemianethesia
C. aphasia
D. dysdiodokokainesia
471. Acute pulmonary edema occurring on opening of the trachea
during tracheostomy is due to:
A. sudden drop of pressure in the tracheobronchial tree and lungs
B. sudden wash of carbon dioxide
C. injury of the esophagus
D. non of the above
472. Hump nose is managed by:
A. Reduction rhinoplasty
B. Augmentation rhinoplasty
C. Nasal tip reconstruction
D. All of the above
473. Rhinoscleroma is treated by the following except:
A. Streptomycin
B. Rifampicin
C. Radiotherapy
D. Puncture and lavage
568. 474-Apnea immediately after tracheostomy is treated by:
A. Multiple skin incisions
B. Inhalation of oxygen
C. Temporary closure of the tracheostomy opening
D. Intubation
475. Kernig?s sign is present in:
A. Otitic meningitis
B. Otitic brain abscess
C. Otitic hydrocephalus
D. All of the above
476. The main site of ossicular necrosis in chronic suppurative otitis
media is:
A. Incudomalleolar joint
B. Long process of incus
C. Stapedial arches
D. Handle of malleus
477. On tracheostomy, apnea developed after incising the trachea is
due to:
A. Sudden wash of CO2 in the trachea
B. Sudden release of the pressure in the trachea
C. Reflex vagal stimulation
D. Non of the above
478. The most common cause of esophageal stricture in children is:
A. Esophageal carcinoma
B. Diphtheria
C. Achalasia of the cardia
D. Corrosive intake
479. Fossa of Rosenmullar is a common site for:
A. Angiofibroma
B. Lipoma
C. Adenoid
D. Nasopharyngeal carcinoma
569. 480-Early sign of nasopharyngeal carcinoma is:
A. Unilateral secretory otitis media
B. Trotter?s triad
C. Petrositis
D. Nasal obstruction
481. All of the following are removed in radical neck dissection EXCEPT:
A. sternocleidomastoid muscle
B. external carotid artery
C. internal jugular vein
D. spinal accessory nerve
E. submaxillary gland
482. Regarding laryngeal cancer, which of the following is FALSE?
A. hoarseness appears early
B. involved nodes are not palpable in 35% of cases
C. distant metastasis appears early
D. direct extension is common
E. it is 90% five-year curable when limited to one cord
483. Which of the following factors is NOT associated with squamous
cell carcinoma
of the larynx?
a) male sex
b) age in fifth and sixth decades
c) history of woodworking
d) large ethanol intake
e) tobacco smoking
484. In LeFort I fractures, the fragment consists of all of the following
EXCEPT
a) upper teeth and palate
b) lower portions of the pterygoid processes
c) portions of the walls of both maxillary antra
d) nasal spine
e) bridge of the nose
570. 485-In general, traumatic perforations of the tympanic membrane:
a) are a surgical emergency
b) will heal spontaneously in most cases
c) usually require operative repair
d) require microsurgical repair
e) require a graft for repair
486. The MOST common organism in acute otitis media of older
children and adults is:
a) Staphylococcus
b) Streptococcus
c) Hemophilus influenzae
d) Klebsiella pneumoniae
e) Pseudomonas
487. Small, malignant tumours of the larynx that are intrinsic in origin
and have not spread beyond the larynx are BEST treated by:
a) irradiation
b) laryngofissure
c) total laryngectomy
d) total laryngectomy and radical neck dissection
e) radium needle implants
488. Mixed tumours of the salivary gland:
a) are most common in the submaxillary gland
b) are usually malignant
c) are most common in the parotid gland
d) usually cause facial paralysis
e) are associated with calculi
489. Which of the following is NOT a cause for conductive hearing loss?
a) otitis media
b) otosclerosis
c) noise-induced hearing loss
d) perforation of the tympanic membrane
e) ossicular chain disruption
571. 490-Conductive hearing losses are usually reversible. Which of the
following conditions is reversible by surgical treatment?
a) otosclerosis
b) presbycusis
c) sudden hearing loss
d) ototoxicity
e) meningitis
491. What is the BEST treatment for most cases of sensorineural
hearing loss associated with aging (presbycusis)?
a) nothing
b) hearing aid
c) ear trumpet
d) diuretic therapy
e) labyrinthectomy
492. The MOST common benign lesion of the external ear is:
A. melanoma
B. chondrodermatitis nodularis chronicus helicus
C. cerumenoma
D. actinic keratosis
E. exostosis of the canal
493. MOST of the infectious and/or inflammatory diseases involving the
middle ear space are secondary to:
A. ciliary dyskinesia
B. resistant pathogens
C. eutstachian tube dysfunction
D. tobacco abuse
E. allergic diathesis
494. Acute otitis is:
A. a rare condition
B. the most common reason ill children visit the doctor
C. usually not accompanied by pain and fever
D. caused by coliform bacteria
E. treated by placing ventilating tubes
572. 495-The MOST common cause for infant stridor, accounting or 60% of
the cases, is:
A. subglottic hemangioma
B. vocal cord paralysis
C. laryngomalacia
D. congenital webs
E. laryngeal cleft
496. What is the MOST common cause of acquired subglottic stenosis?
A. motor vehicle trauma
B. prolonged endotracheal intubation
C. chronic bronchitis
D. tracheoesophageal fistula
E. previous tracheal surgery
497. A 5-year-old child has persistent serous effusions in both ears for
6. months after a routine acute infection. He has a 40-dB condutive
heraring loss in bothears and has been having trouble in school. What
would be the BEST treatment for this child?
A. observe the child for another 3 months
B. prescribe amoxicillin for 10 days
C. recommend hearing aids
D. place ventilating tubes
E. prescribe prophylactic antibiotics for 3 months
498. The following clinical entities are common causes for tinnitus
EXCEPT:
A. high-frequency hearing loss
B. Ménière?s disease
C. ototoxic drugs
D. loud noise exposure
E. acute otitis media
499. Vertigo is very common in all of the following conditions EXCEPT:
A. vestibular neuritis
B. Ménière?s disease
C. presbycusis
D. viral labyrinthitis
E. benign paroxysmal positional vertigo
573. 500-The fastest, safest means of establishing a surgical airway is:
A. endoscopic intubation
B. tracheotomy under local anesthesia
C. tracheotomy under general anesthesia
D. cricothyrotomy
E. puncture through the thyroid membrane
501. The most common cause of unilateral proptosis in adults is:
A. orbital pseudotumour
B. Graves ophthalmopathy
C. paranasal isnus tumours
D. lymphomas
E. meningioma
502. The most common cause of unilateral proptosis in children is:
A. dermoid
B. orbital cellulitis
C. rhabdomyosarcoma
D. lymphangioma
E. capillary haemangioma
503. The most common otolaryngologic manifestation of Wegener?s
granulomatosis at onset is:
A. subglottic stenosis
B. septal perforation
C. hearing loss
D. sinusitis
E. otitis media
504. Excision of the vocal cord and a segment of underlying thyroid
cartilage with or without the ipsilateral arytenoid. Which of the
following is the best description of such maneuver?
A. laryngofissure with cordectomy .
B. supracricoid laryngectomy
C. vertical partial laryngectomy .
D. vestibulectomy
E. cordectomy
574. 505-The proper placement of a Teflon injection in the vocal fold is:
A. as far lateral as possible
B. in Reinke?s space
C. superior to level of the glottis
D. inferior to the level of the glottis
E. in one mass at the level of the vocal process
506. The most common cause of vocal cord paralysis in the adult is
A. surgical trauma
B. Malignancy
C. neurological disorder
D. Idiopathic
E. infections
507. A patient with gastroesophageal reflux a lesion of the middle
portion of the true vocal cord. This finding most likely suggest:
A. Reinke?s oedema
B. contact ulcer
C. Granuloma
D. Carcinoma
E. glottic stenosis
508. A 28 year old patient is diagnosed as having an intracordal vocal
fold cyst. The operative technique of choice is:
A. needle aspiration
B. steroid injection
C. marsupillization
D. vaporization with CO2 laser
E. excision with mucosal preservation
509. Which of the following vocal cord lesions is most appropriate for
CO2 laser excision?
A. polyp
B. nodule
C. intracordal cyst
D. sulcus vocalis
E. papilloma
575. 510-Intubation granuloma can be seen after endotracheal intubation.
On indirect laryngoscopy, the lesion is seen in the following anatomic
position on the vocal cords
A. anterior commisure
B. junction of anterior 1/3, posterior 2/3
C. mid vocal cords
D. posterior 1/3
E. vocal process of the arytenoids
511. A patient referred for evaluation of vertigo has slurred speech.
Speech evaluation reveals ataxic dysarthria. The diagnosis most likely :
A. parkinsonism
B. cerebral palsy
C. brainstem stroke
D. myasthenia gravis
E. multiple sclerosis
512. The most accurate diagnosis of a superior laryngeal nerve
paralysis is made by:
A. - history
B. - physical examination
C. - electroglottography
D. - electromyography
E. - phonoanalysis
513. Bilateral recurrent laryngeal nerve paralysis as the result of
external trauma is most commonly associated with:
A. cricotracheal separation
B. thyroid cartilage fracture
C. arytenoid cartilage dislocation
D. epiglottic prolapse
E. anterior commissure avulsion
514. The sensitivity of a test is defined by which one of the following?
A. true positive/ (true positive + false positive)
B. true positive/ (true positive + false negative)
C. true negatives/ (true negatives + false positive)
D. true negatives/ (true negative + false negative)
E. true positives/ (true negatives + false positives)
576. 515-The specificity of a test is defined by which one of the following?
A. true positive/ (true positive + false positive)
B. true positive/ (true positive + false negative)
C. true negatives/ (true negatives + false positive)
D. true negatives/ (true negative + false negative)
E. true positives/ (true negatives + false positives)
516. Of the following complications of parotidectomy, the most common
is
A. Haematoma
B. Seroma
C. flap necrosis
D. infection
E. salivary fistula
517. What is the most characteristic presentation of sarcoidosis?
A. lymphadenopathy
B. facial paralysis
C. airway obstruction
D. characteristic chest X-ray
E. parotid swelling
518. Distant metastasis are most likely to occur with which of the
following parotid malignancy?
A. acinous cell carcinoma
B. malignant oncocytoma
C. high grade mucoepidermoid carcinoma
D. adenocarcinoma
E. adenoid cystic carcinoma
519. Which of the following factors increases the likelihood of
pleomorphic adenoma recurrence following tumour resection and
postoperative radiation therapy for recurrent tumour?
A. patient age
B. presence of gross tumour following surgical resection
C. total delivered radiation dose
D. number of previous recurrences
E. number of radiation fractions per day
577. 520-Treatment of recurrent pleomorphic adenoma of the parotid
should include:
A. radiation therapy
B. tumour enucleation
C. observation until the tumour becomes symptomatic
D. tumour resection, removal of residual gland and facial nerve
preservation
E. enbloc resection of the tumour with surrounding tissue including the
facial nerve with immediate grafting
521. The presence of lung metastases in a patient with a history of
adenoid cystic carcinoma of the head and neck is associated with:
A. an average survival of less than one year
B. an average survival of greater than 2 years
C. an average survival shorter than with metasteses in other
organs
D. long term cure with a regimen of chemotherapy and radiartion
therapy
E. regional lymph node disease
522. Of the following, the most common primary sarcoma of the major
salivary glands is:
A. malignant fibrous histiocytoma
B. leiomyosarcoma
C. neurosarcoma
D. angiosarcoma
E. osteosarcoma
523. The most common therapeutic option in the management
of patients with carcinoma of unknown primary is:
A. radiotherapy
B. modified neck dissection
C. chemoradiotherapy
D. radiotherapy with salvage surgery
E. selective neck dissection
578. 524-Of biopsy-proven granulomatous inflammation of a cervical lymph
node in a child, the most common cause is:
A. Mycobacterium tuberculosis
B. S. aureus
C. anaerobes
D. cat-scratch disease
E. non-tuberculous mycobacteria
525. A barium swallow of a patient who experienced a sensation of
food sticking, heartburn and episodic choking reveals a marked
cricopharyngeal impression. Which of the following steps would be
most helpful in making a diagnosis?
A. Upper oesophageal sphincter manometry
B. 24-hour ambulatory pH monitoring
C. Pharyngeal plexus block
D. Trial of H2 blockers
E. oesophagoscopy
526. What is the correct treatment of an asymptomatic Zenker?s
diverticulum detected accidentally on a barium study?
A. diverticulectomy
B. diverticulopexy
C. observation
D. cricopharyngeal myotomy
E. Dohlman procedure with CO2 laser
527. A 51-year-old male is sent for a barium swallow &the report comes
back as a peristalsis, esophageal dilatation, and failure of the
lower oesophageal sphincter to relax with retention of ingested
material.This patient probably has:
A. achalasia of the cardia
B. diffuse esophageal spasm
C. oesophageal carcinoma
D. pharyngeal pouch
E. trachea-oesophageal fistula
528. The most common cause of a platelet-related clotting disorder is
A. Idiopathic thrombocytopenic purpura.
B. Megaloblastic anemia
C. Von Willebrand?s disease
D. Aspirin therapy
579. 529-The inner ear has become damaged in the following diseases
except
A. Autoimmune inner ear disease
B. Relapsing polychondritis
C. Mixed connective tissue disease.
D. Polyarteritis nodosa
530. Which of the following lung capacities cannot be measured by a
spirometer?
A. Total lung capacity
B. Inspiratory capacity
C. Vital capacity
D. Functional residual capacity.
531. Which of the following statements is false?
A. OSAS has no effect on intellectual performance.
B. OSAS is more common in males than females.
C. OSAS is associated with increased incidence of myocardial infarction
and stroke.
D. Diminished pharyngeal muscle tone will aggravate symptoms of
snoring and sleep apnea.
E. The most frequently occurring daytime symptom of OSAS is
sleepiness.
532. All of the following are amide local anesthetics except
A. Lidocaine
B. Bupivacaine
C. Ropivacaine
D. Tetracaine.
E. Mepivacaine
533. Which of the following drugs is most likely to cause permanent
hearing loss?
A. erythromycin
B. quinine
C. cisplatin.
D. penicillin G
580. 534-In an otherwise healthy patient with advanced laryngeal cancer,
the treatment regimen that is most likely to result in preservation of the
larynx at 2 years is
A. concurrent chemotherapy and radiation therapy
B. induction chemotherapy followed by radiation.
C. radiation followed by adjuvant chemotherapy
D. total laryngectomy followed by radiation
E. radiation alone
535. An example of a gene identified as playing a role in the molecular
generation of head and neck cancer is
A. Tumor necrosis factor _ (TNF-_)
B. Tubulin
C. p53
D. DNA polymerase
536. Which of the following is an infectious etiology of
laryngotracheobronchitis, or ?croup??
A. Haemophilus influenzae
B. Staphylococcus aureus
C. Branhamella catarrhalis
D. Parainfluenza type 3
E. Varicella
537. The narrowest portion of an infant?s upper airway is
A. The carina
B. The vocal cords
C. The subglottic space
D. The nasal airway
E. The pharynx
538. A patient is noted to have a draining sinus tract deep in the
external canal. Surgical resection will include a dissection of
A. The jugular vein
B. The superficial temporal artery
C. The hyoid bone
D. The stapes
E. The facial nerve
581. 539-The central portion of the hyoid bone should be removed when
removing a
A. Dermoid
B. Thyroglossal duct cyst
C. Thyroid nodule
D. Branchial cleft cyst
E. Thymic cyst
540. The removal of a second branchial cleft sinus tract may require
removal of
A. The thyroid gland
B. The tonsil
C. The parotid gland
D. The hypoglossal nerve
E. The hyoid bone
541. Which of the following types of radiographic evaluations may be
used to evaluate patients with airway noise?
A. Barium swallow study
B. CT
C. MRI
D. Ultrasound
E. All of the above
542. The p53 gene encodes for a protein that
A. Enhances hearing acuity
B. Increases tumor metastases
C. Arrests the cell cycle at G1 in a cell with damaged DNA
D. All of the above
543. Usher?s syndrome type 1 affects
A. The visual system only
B. The auditory system only
C. The vestibular system only
D. All of the above
544. The dorsal (pars superior) portion of the otocyst forms
A. The cochlea
B. The bony portion of the labyrinth
C. The semicircular ducts and the utricle
D. The ossicular chain
E. The tympanic membrane
582
545. Rubella infection of the inner ear can result in
A. Scheibe-type dysplasia of the cochlea and saccule
B. Agenesis of the stria vascularis
C. Malformation of the anterior crista
D. Agenesis of a cochlear duct
E. Duplication of the lateral semicircular duct
546. Von Hippel-Lindau disease may cause otologic symptoms
mimicking
A. Meniere?s disease
B. Benign positional vertigo
C. Presbycusis
D. Sudden idiopathic deafness
547. The appropriate behavioral assessment procedure for a typically
developing 12-month-old infant is
A. Visual reinforcement audiometry
B. Play audiometry
C. Immittance audiometry
D. Otoacoustic emissions
548. For evaluation of tinnitus, the optimal imaging modality is
A. Computed tomographic (CT) scan with intravenous (IV) contrast
B. Positron emission tomographic (PET) scanning
C. Angiography
D. T2-weighted magnetic resonance imaging (MRI)
E. MRI with IV contrast
549. Temporal bone trauma causing facial nerve paralysis is best
evaluated by
A. Surgical exploration
B. CT scan with IV contrast
C. MRI
D. CT scan without contrast high resolution
550. Penetration of which structure allows infection to spread from the
paranasal sinuses to the orbit?
A. Basal lamella
B. Fovea ethmoidalis
C. Uncinate process
D. Lamina papyracea
583. 551-What is a postnatal derivative of the first branchial cleft?
A. The footplate of the stapes
B. The external acoustic meatus
C. The cervical sinus (of His)
D. The incudomalleolar joint
E. The middle ear cavity
552. What is the nerve that innervates the muscle derived from the
third branchial arch?
A. The trigeminal nerve
B. Cranial nerve VII
C. The glossopharyngeal nerve
D. The hypoglossal nerve
E. The recurrent laryngeal branch of the vagus nerve
553. Most acoustic neuromas arise from which of the following nerves?
A. Superior vestibular nerve
B. Acoustic portion of the eighth nerve
C. Inferior vestibular nerve
D. Nervus intermedius
554. Which of the following is not a contraindication for a patient to
undergo magnetic resonance imaging (MRI) scanning?
A. Titanium reconstruction plates
B. Cardiac pacemaker
C. Intracranial aneurysm clip
D. Cochlear implant
555. The best study to evaluate for metastatic nodal disease in the
neck is
A. Ultrasound
B. MRI scan
C. Computed tomographic (CT) scan
D. Plain films
584. 556-The most specific sign of metastatic disease within a lymph node
is
A. Central necrosis
B. Oblong shape
C. Enhancement
D. High signal on T1-weighted image
557. Laryngeal stroboscopy is the least likely to benefit patients who
have:
A. spasticity
B. nonvibrating segment
C. intracordal mass lesion
D. early vocal cord cancer
E. vocal cord at different level
558. a patient with Gastroesophageal reflux has a lesion of the middle
portion of the true vocal cord. This finding most likely suggests:
A. reinke?s edema
B. glottic stenosis
C. contact ulcer
D. granuloma
E. carcinoma
559. In patients with spastic dysphonia, botulinum toxin works by:
A. an unkown mechanism.
B. preventing calcium-dependent release of acetylcholine.
C. increasing calcium-dependent release of acetylcholine
D. increasing level of dopamine in the centeral nervous system.
E. decreasing level of dopamine in the centeral nervous system
560. One stage laryngotracheoplasty for repair of subglottic stenosis in
children avoids all of the following postoperative sequelae except
A. multiple procedures
B. ongoing tracheotomy care
C. need for prologed antibiotic therapy
D. problems of prolonged indwelling laryngeal stent
E. pulmonary atlectasis and neuromuscular weakness
585. 561-Increased strain in muscles used for vocalization may be
associated with all of the following except:
A. asthma
B. bruxism
C. beta adrenergic blockade
D. ventral abdominal hernia
E. sensorineural hearing loss
562. Which of the following anatomic sites of the respiratory tract is at
least risk for the development of recurrent papillomatosis?
A. mid area of the laryngeal surface of the epiglottis
B. nasopharyngeal surface of the soft palate
C. midthoracic tracheobronchial tree
D. false vocal cords
E. true vocal cords
563. Which of the following factors is the most associated with a
decreased likelihood of complication with bronchoscopic removal of a
foreign body?
A. type of foreign body
B. location of foreign body
C. use of telescopic -guided forceps
D. administration of halothane anesthesia
E. bronchoscopic experience of the surgeon
564. A patient undergoing videostroboscopy has normal vocal fold
mucosal waves. This finding is most consistent with a diagnosis of:
A. tuberculosis of the larynx
B. cricoarytenoid joint ankylosis
C. neurogenic vocal fold paralysis
D. submucosal fibrosis of the vocal fold
E. invasive squamous cell carcinoma of the vocal fold
565. A patient referred for evaluation of vertigo has slurred speech.
Speech evaluation reveals ataxic dysarthria. The diagnosis most likely
is:
A. Parkinsonism
B. cerebral palsy
C. brainstem stroke
D. multiple sclerosis
E. myasthenia gravis
587. 571-Which is of the following antihistamines has the longest half-life?
A. astemazole
B. loratidine
C. terfenadine
D. diphenhydramine
E. chlorpheniramine
572. Which of the following symptoms is least likely to be associated
with a cluster headache?
A. Unilateral nasal congestion and rhinorrhea
B. reddening and tearing of one eye
C. sever uniocular pain
D. extreme restlessness
E. nausia and vomiting
573. the most common site of occurrence of nasal polyps is the:
A. nasal mucosa
B. anterior ethemoid
C. posterior ethemoid
D. inferior turbinate
E. osteomeatal region
574. patients who have the highest serum level of bacteria specific IgE
are likely to have:
A. asthma
B. nasal polyosis
C. allergic rhinitis
D. atrophic rhinitis
E. sphenoid sinusitis
575. After endoscopic sinus surgery significant postoperative bleeding
following removal of packing is usually best controlled by:
A. cautery
B. embolization
C. turbinate resection
D. repaking the nose
E. transantral maxillary artery ligation
588. 576-when injury to the lacrimal system is suspected during endoscopic
sinus surgery, the surgeon should :
A. immediatly perform a dacrocystogram
B. perform an external dacrocystorhinostomy
C. perform an endoscopic dacrocystorhinostomy
D. place silastic tubing stents in the lacrimal drainage system
E. completes the procedure, taking care to avoid additional injury
577. Intranasal corticosteroid treatment of allergic rhinitis may have all
of the following effects except:
A. decrease rhinorrhea
B. decrease nasal congestion
C. decrease subjective symptoms of asthma
D. decrease bronchial sensitivity to metacholine challenge
E. increased peak expiratory flow rates
578. the most common cause of recurrent epistaxis after diagnostic
angiography and embolization of the internal maxillary artery is:
A. incomplete embolization
B. recanalization of the artery
C. systemic bleeding diathesis
D. inability to locate the bleeding site
E. continued bleeding from the ethmoid arteries.
579. the most important test in the definitive diagnosis of a 4 year old
child with nasal polyposis and frequent refractory sinusitis is:
A. sweat chloride
B. biopsy of polyp
C. HIV antibody titer
D. pulmonary function
E. quantitative immunoglobulin
589. 580-Absolute bone conduction is reduced in
(a) Otosclerosis
(b) Otitis media with effusion
(c) Endolymphatic hydrops
(d) Traumatic ossicular dislocation
581. Most useful test to diagnose ossicular discontinuity is
(a) Evoked response audiometry
(b) Impedance audiometry
(c) Pure-tone audiometry
(d) Rinne test
590. 582-Which operation is indicated in coalescent
mastoiditis?
(a) Schwartz mastoidectomy
(b) Bondy's operation
(c) Fenestration operation
(d) Combined approach
583. Blue drum is seen in
(a) Acute otitis media
(b) Congenital cholesteatoma
(c) Glomus tumour
(d) Tympanosclerosis
584. Trismus in peritonsillar abscess is due to spasm of
(a) Superior constrictor muscle
(b) Medial pterygoid
(c) Masseter
(d) Temporalis muscle
591. 585-Body of incus is lodged in
(a) Epitympanum
(b) Mesotympanum
(c) Hypotympanum
(d) Protympanum
586. Rehabilitation of the potential for voice after
laryngectomy includes all except
(a) Electrolarynx
(b) Tracheo-oesophageal puncture and voice
prosthesis
(c) Oesophageal voice
(d) Tracheostomy tube with a speaking valve
587. Facial nerve supplies all of the following muscles
except
(a) Stapedius
(b) Anterior belly of digastric
(c) Buccinator
(d) Stylohyoid
592. 588-Trautmann's triangle is a landmark for
(a) Mastoid antrum
(b) Facial recess
(c) Posterior cranial fossa
(d) Endolymphatic sac
589. Best pre-operative measure to reduce blood
loss at surgery in angiofibroma is
(a) Hormonal therapy
(b) Pre-operative radiotherapy
(c) Embolisation
(d) Cryotherapy
590. Type of tympanoplasty in which graft is placed
directly on the head of stapes is called
(a) Type II
(b) Type III
(c) Type IV
(d) Type V
593. 591-Preferred incision for a radical neck dissection
in an irradiated patient is
(a) Haynes-Martin
(b) Mc Fee
(c) Schobinger
(d) Sorenson
592. Which of the following is primarily affected
in Meniere's disease?
(a) Middle ear
(b) Cochlea
(c) VIIIth cranial nerve
(d) Tubal function
593. Which of the following sites is the origin of
laryngocele
(a) Aryepiglottic fold
(b) Laryngeal saccule
(c) Ventricle of Morgagni
(d) Vestibular fold
594. 594-Gradenigo's syndrome is characterized by all
of the following except
(a) Retrobulbar pain
(b) Paralysis CN VI
(c) Spread of middle ear infection to petrous
apex
(d) Cavernous sinus thrombosis
595. Bilateral abductor paralysis is treated by
(a) Thyroplasty type I
(b) Injection of gelfoam into the cord
(c) Lateralisation of arytenoid
(d) Injection of fat into the cord
596. Mickulitz cell is seen in
(a) Rhinoscleroma
(b) Rhinosporidiosis
(c) Salivary gland disease
(d) Thyroiditis
595. 597-Decreased lacrimation on Schirmer's test
indicates a lesion of CN VII
(a) Proximal to geniculate ganglion
(b) In tympanic segment
(c) Vertical segment above the nerve to
stapedius
(d) Vertical segment above the origin of chorda
tympani
598. Meniere's disease is characterised by all
of the following except
(a) Low-pitched tinnitus
(b) Stabbing pain in the ear
(c) Intolerance to loud sounds
(d) Positive Rinne test
599. A 40-year-old man presented with severe attacks
of vertigo. He had history of upper respiratory infection
a week before. No complaints of hearing loss, tinnitus
or facial paralysis. Likely diagnosis in this case is
(a) Benign paroxysmal positional vertigo
(b) Vestibular neuronitis
(c) Meniere's disease
(d) Serous labyrinthitis
596. 600-surgical emphysema after trachestomy is corrected by:
a)taking more stitches of the wound
b)cold compresses
c)widening of the wound by removal of some stitches
d)antihistaminic intake.
601. 60 years old male smoker presents with a unilateral, asymmetric
nodular neck mass. gave him a 2 week trial of antibiotics without
effect.What should be done first?
A ? patient should be followed for an additional 2 weeks
B ? physical exam followed by FNA of the node
C ? open incisional biopsy
D ? schedule patient for modified neck dissection.
602. If neck biopsy occurs prior to definitive treatment which of the
outcomes are worse with metastatic cervical carcinoma?
A. wound necrosis
B. regional neck recurrence
C. distant metastasis
D. all of the above
603. During general anesthesia for a parathyroidectomy for relief of
hyperparathyroidism, you should monitor the ECG closely for?
A Prolonged QT Interval
B. Shortened QT interval
C. Shortened PR Interval
D. None of the above
604. You are performing a general anesthetic for a patient undergoing
dissection of the lower neck for malignancy when you notice a 20% drop
in oxygen saturation, diminished ECG amplitude, a drop in blood
pressure, and diminished breath sounds. Based on these findings, you
suspect?
A. Inadvertent migration of the endotracheal out of the trachea and into
the pharynx
B. Pulmonary embolus
C. Pneumothorax
D. Malfunction of the anesthesia monitors
597. 605. Which induction agent is the best choice for the moderate
hypothyroid patient?
A. Propofol
B. Thiopental
C. Etomidate
D. ketamine.
606. The histopathologic finding associated with the greatest degree of
residual/ recurrent conductive hearing loss after stapedotomy is:
A. Resorptive osteitis of the incus.
B. Prosthesis abutting the bony margin of the oval window.
C. New bone formation in the oval window.
D. Presence of the prosthesis lying on a footplate fragment.
E. Obliteration of the round window by
607. Acute sppurative otitis media associated to facial palsy is adequatly
treated with:
A. Myringotomy and systemic A
B. B- Cortical mastoidectomy with systemic A
B. C-Cortical mastiodectomy with facial n. decompression.
D. Myringotomy with facial n.decompression.
E. Systemic A
B. otosclerosis.
608. Which of the following should raise the greatest suspicion that a
conductive hearing loss on the audiogram is not in fact,conductive at
all?:
A. An absent ipsilateral acoustic reflex.
B. A word understanding score of 64%.
C. Atype A tympanogram.
D. A type Ad tympanogram.
E. Atype C tympanogram.
598. 609-The best investigation to confirm the diagnosis of nasopharyngeal
angiofibroma is
A. biosy
B. CT scan
C. carotid angiography
D. non of the above.
610. The best line of treatment of uncontroled unilateral Meniere?s
disease with severe vertigo and profound SNHL is?
A. Intratympanic injection of corticosteroids.
B. Intratympanic injection of gentamycin.
C. Labrinthectomy.
D. Vestibular nerve neurectomy.
E. Ultrasound destruction of the semicircular canals.
611. The most common age when a thyroglossal cyst usually present
is?
A. At birth.
B. 5-8 years.
C. 8-15 years.
D. 15-18 years.
E. More than 18 years.
612. The buccinator muscle .all are true except:
a. is attached to both jaws opposite the molar teeth
b. is supplied by the facial nerve
c. is continuous with the superior constrictor of the pharynx
d. is a muscle of mastication
e. is pierced by the parotid duct opposite the second upper molar tooth.
613. The parotid gland .all are true except:
a. extends behind the temporamandibular joint
b. is separated from the submandibular gland by sphenomandibular
ligament
c. receives secrotomotor fibres which emerge from the brain in the
glossopharyngeal nerve
d. when acutely enlarged results in pain sensation being transmitted
along the great auricular nerve
e. has a palpable duct
599. 614-The best line of treatment of an infant with subglottic
harmangioma and mild stridor is?
A. Laser coagulation.
B. Tracheostomy.
C. Systemic steroid.
D. Observation.
E. Surgical excision.
615. What type of frontal cell is characterized by a large single cell
pneumatizing cephalad into the frontal sinus?
A. I
B. II
C. III
D. IV.
616. The Messecklinger approach to sinus surgery is ___ to __
dissection.
A. anterior; posterior
B. posterior; anterior
C. lateral; medial
D. medial; lateral
617. What is the most posterior ethmoid cell?
A. agger nasi.
B. Haller.
C. Onodi.
D. Semon .
618. Which of the following is not a major complication of endoscopic
sinus surgery?
A. orbital hematoma
B. blindness
C. diplopia.
D. periorbital emphysema.
600. 619- Which of the following is not a minor complication of endoscopic
sinus surgery?
A. anosmia
B. hyposmia
C. epistaxis
D. periorbital ecchymosis
620. Revision surgery of the paranasal sinus for persistent maxillary
disease may be attributed to any of the following except
A. inadequate removal of the uncinate process.
B. middle turbinate destabilization and lateralization.
C. overwidening of the natural ostium.
D. creation of the posterior fontanelle ostium.
E. refractory rhinitis.
621. One contraindication to the Caldwell-Luc operation is
A. foreign body in the antrum.
B. mucocele.
C. unerupted teeth in children.
D. antral cysts
622. The technique of removal of the intrasinus septum of the frontal
sinus is termed
A. the Ridell procedure.
B. the Lothrop (Chaput-Mayer) procedure.
C. Killian's procedure.
D. Lynch frontoethmoidectomy.
E. Kummel-Beck trephination.
623. The technique of creating a vascularized anterior frontal sinus
table by preserving periosteal integrity when approaching the frontal
sinus is termed
A. the Ridell procedure.
B. the Lothrop (Chaput-Mayer) procedure.
C. frontal osteopathic flap.
D. Lynch frontoethmoidectomy.
E. Killian's procedure.
601. 624- Which of the following is not a complication of external
ethmoidectomy?
A. cranial nerve II injury
B. lacrimal sac injury
C. supraorbital nerve injury
D. infraorbital nerve injury
E. cerebrospinal fluid leak.
625. Which component of the nasal septum may be bilaminar?
A. quadrangular cartilage
B. vomer
C. perpendicular plate of the ethmoid
D. none of the above
E. all of the above
626. The nasal septum and which of the following make up the nasal
valve angle?
A. lower lateral cartilages
B. head of the inferior turbinate
C. upper lateral cartilages
D. pyriform aperture
E. nasal bones
627. Which of the following is not a complication of septal surgery?
A. palatal hypesthesia
B. cerebrospinal fluid leak
C. airway obstruction
D. intranasal synechiae
E. All are recognized complications.
628. Which of the following cannot reduce the incidence of septal
perforations?
A. avoidance of the submucous resection
B. closure of mucosal flap tears
C. replacement of morselized cartilage
D. application of tight septal splints and packing
E. quilting suture to coapt mucosal flaps and eliminate dead
space
602. 629- Blood supply to the cartilaginous nasal septum is directly via the
A. anterior ethmoid artery.
B. posterior ethmoid artery.
C. labial artery.
D. sphenopalatine artery.
E. overlying mucoperichondrium.
630. The risk of sinonasal cancer is increased with exposure to all of
the following except
A. hydrocarbons.
B. wood particles.
C. thorium dioxide.
D. salted fish.
631. What is the commonly accepted point of origin of juvenile
nasopharyngeal angiofibroma?
A. superior/posterolateral nasal cavity
B. nasopharynx
C. posterior nasal septum
D. superoposterior base of the skull
632. Juvenile nasopharyngeal angiofibroma is classically associated
with all of the following except
A. epistaxis.
B. nasal obstruction.
C. retrobulbar pain.
D. adolescent boys
633. Which of the following statements regarding nasal cavity
malignant melanoma is true?
A. The probability of death does not decrease with time as it
does with other tumors.
B. Cure rates are high.
C. Incidence of local recurrence is low.
D. It originates more commonly in the sinuses than in the nasal
cavity.
603. 634-What are the histologic features of esthesioneuroblastoma?
A. well-differentiated cells with abundant cytoplasm
B. poorly differentiated cells with multiple mitotic figures
C. undifferentiated small neuroepithelial cells with densely
staining nucleoli and scant cytoplasm
D. granular cells in various stages of differentiation with a high
mitotic rate
635. Which of the following cochlear components in most sensitive to
acoustic overstimulation?
A. inner hair cells
B. outer hair cells
C. spiral ganglion cells
D. dendritic endings of the cochlear nerve
636. The type of hearing loss caused by sound overexposure with the
most insidious nature is
A. temporary threshold shift.
B. acoustic trauma.
C. noise-induced hearing loss.
D. all of the above
637. The frequency region first affected by habitual exposure to
excessive sound is typically ________ kHz.
A. 0.5 to 2
B. 3 to 6
C. 6 to 8
D. 8 to 10
638. Which recently discovered objective measure of cochlear function
promises to be capable of detecting noise-induced hearing loss at an
early stage of development that precedes detection by the pure-tone
audiogram?
A. auditory brain stem response
B. middle-latency evoked response
C. electrocochleography
D. evoked otoacoustic emissions
604. 639- Which of the following has been proposed as the mechanism by
which the sensory cell degenerates following sound overexposure?
A. metabolic exhaustion due to overuse
B. activity-induced ischemia
C. ionic poisoning due to microbreaks in the cytoarchitecture of
the organ of Corti
D. all of the above
640. Which of the following components of the auditory pathway is
suspected of contributing to the susceptibility of an individual to sound
overexposure effects?
A. cochlear efferent system
B. medial geniculate
C. auditory cortex
D. middle ear
641. Which of the following agents interacts with loud sounds
synergistically to produce noise-induced hearing loss?
A. some medicinal drugs
B. some industrial chemicals
C. segmental or whole-body vibration
D. all of the above
642. What noise level (in dB
A. requires industries to implement a
hearing-conservation program?
A. 80
B. 85
C. 90
D. 100
643. Which entity holds that if one sound contains half as much energy
as a second sound, but lasts twice as long, both sounds are capable of
producing the same amount of damage to the ear?
A. dBA scale
B. linear decibel scale
C. distortion-product otoacoustic emissions
D. equal-energy principle
605. 644- What role does the otolaryngologist play in dealing with noise-
induced hearing loss?
A. diagnose the condition
B. counsel patients on preventive measures
C. provide medicolegal testimony in workmen's compensation
cases
D. all of the above
645. What is the most common site of fixation of the stapes?
A. annular ligament
B. posterior crus
C. anterior crus
D. entire footplate
646. At times early stapes fixation of otosclerosis can be diagnosed
A. by seeing stapes fixation on physical examination.
B. by seeing an abnormality of the tympanic membrane.
C. when a maximum conductive hearing loss is present on
audiogram.
D. when an on-off stapedial reflex can be seen on impedance
audiometry
647. The modern stapedectomy was first described in 1956 by
A. Toynbee.
B. Rosen.
C. Shea, Jr.
D. House.
648. Measurement of the distance from the incus to the footplate is
important in order to determine
A. if stapes fixation is the cause of hearing loss.
B. the proper length of prosthesis needed to avoid damage to
the contents of the vestibule.
C. the type of prosthesis required.
D. all of the above
606. 649-A major advantage of using a microdrill is that it
A. creates a perfect round 0.7-mm fenestra.
B. is easier to use than a laser.
C. is atraumatic.
D. Any type of prosthesis can be used
650. Which of the following is an advantage of the modified radical
mastoidectomy over the intact wall procedure?
A. the ability to maintain the normal anatomy of the external ear
canal
B. the ability to easily identify recurrent cholesteatoma
C. fewer episodes of postoperative otorrhea
D. superior hearing results
E. the fact that long-term follow-up is not necessary
651. Surface landmarks used for cortical mastoidectomy include the
A. sinodural angle of Citelli.
B. petrosquamous (Korner's) septum.
C. temporal line.
D. scutum.
E. Ponticulus .
652. The tegmen tympani
A. separates the mastoid air cells from the middle fossa.
B. lies just medial the descending facial nerve and is difficult to
visualize directly.
C. is also known as the posterior fossa dural plate.
D. is the mastoid cortex landmark that corresponds to the
mastoid antrum.
E. is an important landmark for locating the facial nerve
653. The facial recess provides access to the
A. internal auditory canal.
B. retrolabyrinthine air cells.
C. epitympanic space.
D. posterior mesotympanum.
E. mastoid antrum
607. 654-Careful opening of the subarcuate cell tract
A. will allow the surgeon to clean out the sinus tympani.
B. is an alternate route to the antrum.
C. allows drainage of the petrous apex.
D. will expose the internal auditory canal.
E. exposes the horizontal facial nerve .
655. Following a properly performed radical mastoidectomy,
A. there should never be any more episodes of otorrhea.
B. there should be a second-stage ossicular reconstruction
within 12 months.
C. it is relatively easy to reconstruct the posterior canal wall.
D. there is a maximum conductive hearing loss.
E. it is impossible to use a hearing aid on that side.
656. The advisability of extracranial-to-intracranial bypass during
extirpative skull-base surgery for lesions involving the internal carotid
artery is best determined by
A. physical findings.
B. anamnestic information.
C. cerebral angiography.
D. magnetic resonance angiography.
E. the xenon?computed tomography balloon occlusion test.
657. Which of the following skull-base surgical approaches provides
direct access to the petrous apex and infralabyrinthine areas?
A. infratemporal
B. midface degloving
C. basal subfrontal
D. anterior craniofacial
E. extended maxillotomy
658. For tumors involving the middle cranial fossa and the more inferior
portions of the infratemporal fossa, the best approach is the
A. lateral facial.
B. lateral transtemporal sphenoid.
C. transparotid.
D. midface degloving.
E. transmandibular.
608. 659-For large lesions of the clivus that extend into the parapharyngeal
space, adequate exposure is obtained by
A. transethmoidal?sphenoidotomy.
B. transseptal?sphenoidotomy.
C. lateral rhinotomy.
D. transantral.
E. transoral with mandibulotomy.
660. Good exposure of the ethmoid block is obtained by
A. lateral rhinotomy.
B. transseptal sphenoid.
C. transpalatal.
D. transoral.
E. Mandibulotomy
661. Which of the following is not an indication for open rhinoplasty?
A. severe asymmetry of the lateral bony walls
B. congenital anomalies
C. use of a columellar strut
D. extremely wide bony or cartilaginous hump
E. nasal deformity that is difficult to assess
662. The fascial graft technique
A. is best accomplished through an open rhinoplasty approach.
B. softens implant irregularities.
C. uses temporalis fascia or synthetic sheeting.
D. has minimal donor site morbidity.
E. all of the above.
663. Which of the following statements is false?
A. In a previously operated nose, mucoperiostial flap elevation
should begin at the maxillary crest and proceed upward.
B. Release of septal angulations and fibrous contractions is
important in septal framework surgery.
C. In correction of septal curvature, cartilage wedges are
removed from the concave side of the septum while cartilage
scoring is used on the convex side.
D. Most angulations and deviations of the septum occur at the
bony?cartilaginous junction
609. 664-Major characteristics of the unilateral cleft-lip nasal deformity do
not include
A. ipsilateral retrodisplacement of the lower lateral cartilage.
B. lateral and inferior displacement of the ipsilateral alar base.
C. ipsilateral maxillary underdevelopment.
D. ipsilateral domal depression .
665. Which of the following statements regarding spreader grafts is
false?
A. Spreader grafts are placed between the septal cartilage and
the lower lateral cartilages.
B. Spreader grafts may be used unilaterally or bilaterally.
C. Auricular or septal cartilages may be used.
D. Spreader grafts are most easily placed via a closed
rhinoplasty approach.
E. Ideally, spreader grafts should be placed submucosally
666. The relationship between the function of the upper and lower
lateral cartilage in the majority of humans is
A. interlocked scroll.
B. overlap.
C. end-to-end contact.
D. opposed scroll.
667. Reflex lacrimal gland secretion is mediated by parasympathetic
fibers of cranial nerve
A. V.
B. VII.
C. III.
D. IX.
668. The sphenopalatine ganglion is
A. a special sensory afferent.
B. sympathetic.
C. parasympathetic.
D. a somatic sensory afferen
610. 669-The nasopalatine nerve supplies sensation to the mucosa of the
A. anterior premaxillary palate.
B. soft palate.
C. lateral nasal passage.
D. anterior cheek.
670. The anatomic boundaries of zone III of the neck are from the
A. hyoid to the base of the skull.
B. superior border of the thyroid cartilage to the base of the skull.
C. hyoid to the mandible angle.
D. mandible angle to the base of the skull.
E. clavicle to the cricoid .
671. Which zone of the neck has the most difficult surgical access?
A. base of the skull region
B. midcervical region
C. lower cervical region below the cricoid
D. posterior neck triangle
E. anterior cervical triangle
672. A 40-year-old man who sustained a stab wound to zone III of the neck is
hemodynamically stable but has an acute hypoglossal nerve paralysis. What is
the next diagnostic step?
A. four-vessel angiography
B. computed tomography
C. magnetic resonance imaging
D. lateral soft tissue of the neck
E. direct laryngoscopy
673. The best incision to explore the carotid sheath for a unilateral penetrating
neck injury is a(n)
A. modified Conley incision.
B. lateral cervical incision along the anterior sternocleidomastoid muscle.
C. MacFee incision.
D. H incision.
E. Schobinger incision
t.
611. 674-What injured area is the most commonly missed when evaluating a
penetrating neck injury during endoscopy?
A. esophageal inlet
B. distal cervical esophagus
C. nasopharynx
D. vallecula
E. supraglottic area
675. Recognised complications of acute suppurative otitis media
include except?
A. Temporal bone abscess.
B. Facial nerve palsy.
C. Abducent nerve palsy.
D. Bartholin abscess.
E. Cholesterol granuloma.
ANSWERS
612. 216:A
217:C
218:B
219:B
220:B
221:C
222:A
223:A
224:C
225:C
226:C
227:C
228: D
229:C
230:C
231:A
232:C
233:A
234:D
235:C
236:D
237:C
238:D
239:C
240:C
241:A
242:C
243:B
244:B
245:B
246:A
247:D
248:C
249;D
250:D
173:B
174:C
175:D
176:A
177:D
178:B
179:D
180:A
181:D
182:A
183:B
184:D
185:B
186:C
187:A
188:B
189:C
190:B
191:D
192:C
193:B
194:B
195:E
196:D
197:D
198:C
199:C
200:C
201:B
202:A
203:C
204:C
205:B
206:D
207:D
208:B
209:B
210:D
211:B
212:D
213:E
214:D
215:B
130:B
131:A
132:B
133:A
134:A
135:B
136:D
137:E
138:A
139:B
140:C
141:B
142:B
143:A
144:D
145:E
146:D
147:B
148:B
149:C
150:B
151:C
152:C
153:B
154:B
155:E
156:A
157:C
158:D
159:C
160:D
161:A
162:E
163:B
164:B
165:C
166:C
167:A
168:C
169:B
170:C
171:C
172:E
87:B
88:B
89:C
90:E
91:C
92:C
93:C
94:D
95:D
96:D
97:A
98:E
99:E
100:A
101:C
102:E
103:E
104:A
105:C
106:C
107:C
108:B
109:C
110:D
111:C
112:C
113:C
114:E
115:C
116:C
117:C
118:A
119:D
120:D
121:D
122:C
123:A
124:C
125:D
126:D
127:C
128:B
129:C
44:B
45:C
46:B
47:B
48:A
49:B
50:B
51:A
52:A
53:A
54:D
55:A
56:C
57:C
58:C
59:C
60:C
61:B
62:C
63:D
64:D
65:E
66:A
67:D
68:B
69:C
70:C
71:A
72:C
73:D
74:C
75:E
76:C
77:A
78:D
79:C
80:A
81:D
82:A
83:D
84:C
85:B
86:C
1:D
2:D
3:E
4:C
5:C
6:E
7:A
8:D
9:D
10:D
11:D
12:A
13:B
14:B
15:E
16:A
17:A
18:A
19:C
20:D
21:A
22:C
23:D
24:A
25:E
26:E
27:E
28:B
29:C
30:C
31:E
32:A
33:B
34:B
35:C
36:D
37:D
38:E
39:E
40:C
41:D
42:A
43:A
613.